Neurology Shelf

Ace your homework & exams now with Quizwiz!

What is the Lhermitte sign?

- Sensory phenomenon seen in Ms in whcih the patient feels an electrical sensation radiating down the spine when the neck is passively flexed

Signs of cavernous sinus thrombosis?

- Severe headache, bilateral periorbital edema, and cranial nerve III, IV, V and VI deficits

What type of gait is characteristic in Parkinson's?

- Shuffling (hypokinetic) gait

What is neuromyelitis optica?

- Signs and symptoms of bilateral optic neuritis + transverse myelitis (paraparesis, bladder and bowel dysfunction, and sensory deficit)

Friedrich ataxia is degeneration in what part of the body? What chromosome?

- Spinal cord - Chromosome 9

Vessels affected in each type of hemorrhage?

- Subdural: bridging veins - Epidural: meningeal artery - Subarachnoid: rupture saccular aneurysm

Person keeps burning hands because they can't feel temperature or pain. What is it?

- Syringomelia - cyst in the central spinal cord

What is epilepsia partialis continua?

- Condition of persistent focal motor seizure activity - in essence a focal motor status epilepticus - Distal hand and foot muscles are most frequently affected

Hemispatial neglect is due to damage to what area of the brain?

- Right parietal lobe

What is the peak time for alcohol withdrawal seizures?

- 1-2 days

Neurofibromatosis 1 vs 2?

- 1: Cafe au lait spot, neurofibromas, optic gliomas, armpit freckling, lisch nodules, bony abnormalities; remember chromosome 17 - 2: think of meningiomas and schwannomas (bilateral acoustic neuromas); remember chromosome 22

What is lathyrism?

- A condition characterized by slow or subacute onset of spastic paraparesis in the setting of excessivec dietary reliance on the chickling pea (L. sativus) - Syndrome typically occurs in epidemics in the setting of famine or war, in which people are forced to rely excessively on this legume

What is blepharospasm?

- A form of focal dystonia - Eye contraction/closing triggered by things such as bright light

First-line medical treatment for IIH?

- Acetazolamide

What is the initial treatment for MG?

- Acetylcholinesterase inhibitor - pyridostigmine

Treatments for each: 1) Acute dystonia 2) Akathisia 3) Parkinsonism 4) Tardive dyskinesia

- Acute dystonia: benztropine, diphenhydramine - Akathisia: beta blocker, benzodiazepine, benztropine - Parkinsonism: benztropine, amantadine - Tardive: valbenazine, deutetrabenazine

Describe malignant hyperthermia and the treatment

- Acute severe fever, tachypnea, tachycardia, and high rigidity after receive halothanes or succinylcholine - Treat with dantrolene

The hypogonadism and anosmia of Kallman syndrome usually attract medical attention during which stage of life?

- Adolescence

Arm hit while abducted and externally rotated. What injury and what finding on exam?

- Anterior shoulder dislocation - weakness on shoulder abduction

What drug has been approved for the treatment of ALS?

- Riluzole - Glutamate inhibitor that is currently approved for use in patients with ALS - Although, it cannot arrest the underlying pathological process, it may prolong survival and the time to tracheostomy

Patient has an intracerebral hemorrhage and is hospitalized. 6 days after being admitted, the patient abrutly loses consciousness and starts having a seizure. What should he be treated with?

- Antiepileptic, Levetiracetam (for example) - Seizures are common after an intracerebral hemorrahge - You want to use a seizure drug you can use at full strength right away - Something like lamotrigine would be a bad choice because it needs to be titrated up slowly

Why can extreme heat stroke cause persistent nose bleeding?

- Complication of heat stroke includes DIC, which will cause persistent bleeding

A type 1 Chiari malformtion usually becomes symptomatic as what in adults?

- Ataxia and other cerebellar symptoms - Type 1 : cerebellar tonsils extend below the foramen magnum - affected persons do not usually become symptomatic until they are adults, and the symptoms are largely referable to the cerebellum - Type 2: cerebellar anatomy is much more deranged, and the cerebellar vermis lies well below the foramen magnum - often become symptomatic at birth or during infancy

Most likely cause of dilated abdominal aorta?

- Atherosclerosis

Protein, glucose, and opening pressure in CSF for bacterial vs. viral causes?

- Bacterial: low glucose, high protein, high opening pressure - Viral: normal glucose, high protein, normal/high opening pressure

Name the nerve that innervates these muscles: - Biceps - Brachioradialis - Triceps - Deltoid - Serratus anterior - Rhomboids - Trapezius - Lats

- Biceps: musculocutaneous nerve (C5-7) - Brachioradialis: radial nerve (C5-8, T1) - Triceps: radial nerve (C5-8, T1) - Deltoid: axillary nerve (C5-6) - Serratus anterior: long thoracic nerve (C5-7) - Rhomboids: Dorsal scapular nerve (C5) - Trapezius: spinal accessory nerve (CN XI) - Lats: Thoracodorsal nerve (C6-8)

Findings in anterior spinal artery syndrome

- Bilateral hemiparesis at the level of the injury and below - Diminished bilateral pain and temperature sensation - Intact bilateral propioception, vibratory sensation and light touch

Neurofibromatosis 2 findings: cause and findings

- Bilateral schwanommas - NF2, chromosome 22

What are the four cardinal features of Parkinsons disease?

- Bradykinesia - Tremor - Rigidity - Postural instability

Branches of the sciatic nerve

- Branches into the common peroneal and tibular - Common peroneal nerve further dividies into deep and superficial peroneal nerves

Ulnar nerve: nerve roots, muscles innervated/what it does, mechanisms of injury, injury result

- C8-T1 - Flexion of the 4th and 5th MCPs, abduction and adduction of the fingers - Injury at elbow (cubital tunnel) or wrist (Guyon canal cyst) - Presence of a claw hand deformity, due to hyperextension of the 4th and 5th MCP and flexion at IP, loss of sensation on 4th and 5th fingers

Which cranial nerve is most likely to be injured in a patient with sarcoidosis?

- CN VII - Facial nerve paralysis is the most common neurological injury associated with sarcoidosis

What imaging is preferred when somebody comes in with signs of a stroke?

- CT of the head without contrast - MRI is technically better, but CT is faster and cheaper

A patient has positive antibodies against the nicotinic receptors on the endplate. What should be your next step in diagnosis?

- CT scan of the chest - You need to rule out a thymoma - B/c the antibodies are there, you don't' need to do something like an Edrophonium test to get your diagnosis of MG

Neurofibromatosis 1: cause and findings

- Cafe au lait spots - Axillary freckling - Lisch nodules - Optic gliomas - NF1, chromosome 17

Why can amitriptyline can dizziness and how can you test for it?

- Can cause orthostatic hypotension because it interacts with the alpha adrenergic receptors - Can test for this as the cause by taking orthostatic BP measurements

What is Dejerine-Roussey syndrome?

- Can happen after a stroke in the thalamus - Severe paroxysmal burning pain over the affected area and is classically exacerbated by light touch (allodynia)

Most common mononeuropathy in dialysis? Presentation

- Carpal tunnel syndrome - Presents with end-stage renal disease on dialysis - Characterized by pain and paresthesia in the lateral hand, symptoms typically worsen during dialysis and are more severe in the arm with vascular access

Atrophy of what in Huntington's disease?

- Caudate nucleus

What is subclavian steal syndrome?

- Caused by severe atherosclerosis of the left subclavian artery proximal to the origin of the vertebral artery - Significant stenosis or occlusion leads to decreased pressure in the distal subclavian artery and reversal ("steal") of blood flow in the ipsilateral vertebral artery - Left subclavian is more commonly affected - Many patient are asymptomatic; symptoms can include symptoms of ischemia in the affected upper extremity such as pain, fatigue, and paresthesias - They might also less commonly develop dizziness, ataxia, and disequilibrium - PE usually shows a significantly lower blood pressure in the affected arm and a systolic bruit just above the clavicle - You may also have a fourth heart sound

What is Von Hippel Lindau syndrome?

- Causes tumors and cysts to grow in the body such as in the kidney and pancreas - Hemangioblastomas in the brain

Affects on the face in central vs peripheral nerve lesions?

- Central: contralateral lower facial weakness, spares the upper forehead muscles - Peripheral: unilateral facial weakness that does not spare the forehead muscles

Intention tremors most likely results from disease in what structure?

- Cerebellum

Explain the "COWS" test.

- Cold water: eyes deviate towards the side, but nystagmus away from the ear - Warm water: eyes deviate away from the ear, but nystagmus towards the ear *Remember, the acronym is describing the direction of nystagmus

What is critical illness polyneuropathy?

- Commonly occurs after a prolonged stay in the ICU, and is characterized by peripheral weakness and reduced reflexes

Non-communicating vs communicating hydrocephalus.

- Communicating: flow of CSF is blocked after leaving the ventricles - Non-communicating: flow is blocked going through the ventricles; most common location is aqueduct of Sylvius which connects the third and fourth ventricles

What is transient global amnesia?

- Complete and reversible anterograde and retrograde memory loss lasting up to 24 hours

Median nerve: nerve roots, muscles innervated, mechanisms of injury, injury result

- Contributions from C5-7 and C8-T1 - Flexion of the 1st-3rd MCP, arm pronation - Injury: supracondylar humeral fracture, entrapment at the level of the elbow, wrist laceration, carpal tunnel syndrome - Inability to flex 1-3rd MCPs, loss of sensation on these fingers

radial nerve: nerve roots, muscles innervated, mechanisms of injury, injury result

- Contributions from C5-8, T1 - Sensation back of hand, extension of hand, wrist, triceps - Saturday night palsy, mid-shaft humeral fracture - Wrist drop, loss of sensation on back on hand

What medication can be beneficial in the treatment of DMD, but not BMD?

- Corticosteroids

Decreased vision in the temporal fields on both sides + headaches. Think what?

- Craniopharyngioma!

What type of reflexes can still be seen in brain death?

- Deep tendon reflexes

Tay Sachs diseases: deficiency of what enzyme, what builds up, features

- Deficiency: beta-hexoaminidase A - Builds up: Ganglioside GM2 - Features: neurodegeneration, cherry red spot on macula

What is multiple system atrophy (Shy-Drager syndrome)?

- Degenerative diseases chaacterized by: 1) Parkinsonism 2) Autonomic dysfunction 3) Widespread neurological signs

VHL: cause and findings

- Deletion of VHL gene, tumor suppressor on chromosome 3 - Hemiangioblastomas - Bilateral renal cell carcinomas - Pheochromocytomas - Renal and pancreatic cysts

CT scan of head with numerous punctate hemorrhages with blurring of the gray-white matter interface. What is it?

- Diffuse axonal injury

What are Argyll Robertson pupils?

- Don't constrict in response to light but do accomadate (ex. looking at a finger)

Describe polymylagia rheumatica.

- Elevated ESR - Anemia - Weight loss - Malaise - Older people

What causes the transient monocular blindness in carotid artery disease?

- Embolism to the central retinal artery or one of its branches

Fabry disease: deficiency of what enzyme, what builds up, features

- Enzyme: alpha-galactosidase A - Builds up: ceramide trihexodide - dysethesias, hypohyrosis, angiokeratomas, cardiomyopathy, nephropathy

Metachromatic leukodystrophy: deficiency of what enzyme, what builds up, features

- Enzyme: arylsulfatase A - Builds up: cerebroside sulfate - Ataxia, dementia

Krabbe disease: deficiency of what enzyme, what builds up, features

- Enzyme: galactocerebrosidase - Builds up: galactocerebrosides - peripheral neuropathy, loss of vision

Gaucher disease: deficiency of what enzyme, what builds up, features

- Enzyme: glucocerebrosidase - Builds up: glucocerebrosides - Hepatosplengomegaly, bone crises, blood cytopenias, Gaucher cells

Niemann Pick disease: deficiency of what enzyme, what builds up, features

- Enzyme: sphingomyelinase - Builds up: sphingomyelin - progressive neurodegeneration, cherry red spot on macula, hepatosplenomegaly

Rye grains - which poisoning?

- Ergots

What two drugs can you use for absence seizures?

- Ethosuximide - first choice - Can cause stomach upset - can use divalproex sodium instead if not tolerated, or if they don't respond to ethosuximide

What is a bad complication of IIH if left untreated?

- Fairly "benign" condition but can lead to blindness if left untreated

Describe the pathway of the DCML tract and what it transmits.

- Fine touch, vibration, propioception -Sends afferent fibers from 2 dorsal columns - gracile fasciculus and cuneate fasciculus - synapse in the gracile and cuneate nuclei in the medulla - Decussate in the medulla and travel up to the thalamus, where they synapse again in the VPL (sensation from neck/truck/extremities) and VPN (sensation from head) - Ascend to posterior limb of internal capsule, synapse in the primary somatosensory cortex

What is Meig syndrome?

- Form of focal dystonia characterized by belpharospasm, forceful jaw opening, lip retraction, neck contractions and tongue thrusting

Huntington's disease causes degeneration primarily of what neurons/neurotransmitter?

- GABA

Treatment for an acute MS exacerbation?

- Glucocorticoids

What four drugs can help with the tics of Tourette syndrome?

- Haloperidol - Pimozide - Trifluoperazine - Fluphenazine

What is the most common symptom is patients with brain abscesses?

- Headache

what is a basilar migraine?

- Headache comes after the neurological deficits - Can have weakness, loss/blurry of vision, and loss of consciousness

Describe Brown Sequard syndrome?

- Hemisection of spinal cord 1) Corticospinal tract: spastic paralysis on the same side of the body below the level of the lesion; at the level of the lesion there is flaccid paralysis 2) Lesion to dorsal column: ipsilateral loss of vibration and propioception, fine touch 3) Lesion to spinothalamic: contralateral loss of pain and temperature sensation below the lesion

Findings of Brown-sequard syndrome?

- Hemisection of the spinal cord - Ipsilateral hemiparesis at level of injury and below - Ipsilateral diminished propioception, vibration, and light touch at level of injury and below - Contralateral diminished pain and temperature 1-2 levels below the injury

Appearance of hemorrhagic vs ischemic stroke on CT?

- Hemorrhagic: appears white/bright - Ischemic: appears dark

What is Leber optic atrophy?

- Hereditary optic neuropathy characterized by bilateral, painless, subacute loss of central vision - Runs in families

Presentation of a lateral medullary stroke (Wallenberg syndrome)?

- Horizontal and rotational nystagmus - Vertigo with falling to the side of the lesion - Loss of temp/pain in ipsilateral face and contralateral body - Ipsilateral bulbar weakness, ipsilateral Horner's syndrome - Usually occlusion of vertebral artery/PICA

What is the strongest risk factor for stroke?

- Hypertension

How does hyperventilation lower ICP?

- Hyperventilation decreases CO2 - Less CO2 = cerebral vasoconstriction = decreased blood flow = decreased ICP

Other than IV hydration, what is the next best step in management of heat stroke?

- Immersion in ice water

What is the main effect of alcohol on a fetal brain?

- Impaired neuronal migration

Patients with MS symptoms often notice their symptoms get worse with what? (environmental condition)

- Increased temperature

A patient has bone metastases from prostate cancer. He continues to have unrelenting pain that is not relieved by NSAIDS. What should you give him next?

- Intermittent doses of short-acting opioids

Manifestations of the following mineral deficiencies: - iron - selenium - Zinc

- Iron: microcytic anemia - Selenium: thyroid dysfunction, cardiomyopathy, immune dysfunction - Zinc: alopecia, pustular skin rash, hypogonadism, impaired wound healing, impaired taste, immune dysfunction

How does chronic noise exposure lead to sensorineural hearing loss?

- Irreversible death of hair cells in the cochlea

Describe Parinaud syndrome?

- Lesion affecting structures in the dorsal midbrain - Upward gaze palsy + convergence retraction nystagmus + light-near dissociation + bilateral lid retraction - PINEAL gland tumor

What are the two main complications after a subarachnoid hemorrhage and how can you tell the difference between them?

- Look at the timing - Re bleeding - major caused of death within the first 24 hours of presentation - Vasospasm - 3-10 days after presentation and is the major cause of morbidity and death

Symptoms of cauda equina syndrome?

- Low back pain - Loss of bowel-bladder control - Numbness around anal area - Pain that radiates down the legs

What is the treatment of choice for agitation from delirium in the elderly?

- Low-dose haloperidol

What organ is the most common source of metastatic tumors to the brain?

- Lung

How does phenytoin toxicity present?

- Manifests as signs of cerebellar dysfunction - horizontal nystagmus, ataxia, dysmetria on finger-nose tesing, slurred speech, and NV - Hyperreflexia may also occur

The tentorium cerebelli is a common site for origin of which brain tumor?

- Meningiomas

Olfactory hallucinations before a seizure are assocaited wtih a lesions where?

- Mesial temporal lobe, particularly the hippocampus or parahippocampal gyrus

What is the most common cause of intractable complex partial seizures in adults?

- Mesial temporal sclerosis - Medications won't really help, but resection of the right anterior temporal lobe maybe allow seizure freedom in up to 80% of cases

Patient with a 45-pack year smoking history. Has a witnessed seizure. Brain MRI shows several, discrete, circumscribed lesions at the junction of the gray and white matter with surrounding lesions. What is this?

- Metastatic lung cancer

What is the most common cell type from which primary brain tumors arise? What is the most common type of primary brain tumor?

- Most commonly arise from glial cells - Most common type: malignant astrocytoma

What changes will you see on brain imaging of a patient with Alzheimer's?

- Most prominent is temporal lobe atrophy

Radial nerve: motor

- Motor: extension at all arm, wrist, and proximal finger joints below the shoulder; forearm supination

Ulnar nerve: motor and sensory

- Motor: finger adduction and abduction other than thumb; thumb adduction, flexion of digits 4 and 5; wrist flexion and adduction - Sensory: skin over digits 4 and 5 on palmar side

Median nerve: motor and sensory

- Motor: thumb flexion and opposition, flexion of digits 2 and 3, wrist flexion and abduction, forearm pronation - Sensory: skin over digits 1, 2, and 3 on palmar side

Sturge Weber Syndrome: cause and findings

- Mutation of GNAQ gene - Port wine stain - Leptomeningeal angioma - Early onset glaucoma - Intellectual disability

Patient has bilateral ptosis. An ice pack in placed on his eyes for 2 minutes, leading to improvement of this ptosis. What is this?

- Myasthenia gravis - The cold decreases the acetylcholinesterase breakdown of acetycholine

Does B12 deficiency always present with a macrocytic anemia?

- NO

In a patient with signs/symptoms of dementia, what other workup should be standard other than a montreal cognitive assessment?

- Neuroimaging - CT or MRI of brain - CBC, B12, TSH and CMP - Only do folate, syphillis, and vitamin D with specific risk factors

Does Methylpredisone affect long-term outcomes with optic neuritis?

- No, outcome at 6 months is unchanged - However, it does speed the recovery

Non-pupil sparing vs pupil-sparing CN III palsies?

- Non-pupil sparing (the affected pupil will be dilated): frequently caused by mass effect and should be considered to be due to an intracranial aneurysm until proven otherwise; immediate MRI or CT of the head - Pupil-sparing: typically caused by microvascular ischemia and are associated wtih diabetes, hypertension and advanced age

Air conduction greater than bone conduction bilaterally. What type of hearing loss?

- Sensorineural (common is presbycusis which is just age-related hearing loss)

Explain the tuning fork on the head for hearing problems and what the findings mean.

- Normal: vibrations from a tuning fork are transmitted through the skull to both ears; bone conduction of sound through the skull should be equal in both ears - Sensorineural hearing loss: patient will hear the midline fork more loudly in the unaffected ear (ex. left sided sensorineural hearing loss - will be louder in right ear) - Conductive hearing loss: vibrations are louder in the affected ear (ex. left sided conductive hearing loss - will be louder in left ear)

What is non-fatiguable nystagmus and what it is concerning for?

- Nystagmus that lasts more than 1 minute and cannot be suppressed by visual fixation - This can be seen with central vertigo, NOT peripheral, and you should get head imaging immediately

What is general paresis?

- One of the manifestations of neurosyphillis - Chronic, often insidious meningoencephalitis that may be delayed up to 20 years after the original infection - manifests as dementia, delusions, dysarthria, tremor, myoclonus, seizures, spasticity and Argyll Robertson pupils

What nerve controls corneal sensation?

- Opthalamic branch of the trigeminal nerve (CN V)

Explain each of the visual field defects: - Lesion of optic nerve - Lesion at optic chiasm - Lesion at optic tract - Lesion at parietal upper optic radiation - Lesion at temporal lower optic radiation - Lesion at occipital visual cortex

- Optic nerve: ipsilateral monocular visual loss - Optic chiasm: bitemporal hemianopia - Optic tract: contralateral homonymous hemianopia - Parietal upper optic radiation: contralateral homonymou inferior quadrantopia - Temporal lower optic radiation: contralateral homonymous superior quadrantopia - Occipital visual cortex: contralateral homonymous hemianopia with macular sparing

"Washed-out" color vision?

- Optic neuritis

Explain the tract of the lateral corticospinal tract.

- Originates in the precentral gyrus (primary motor cortex) - Axons then travel in the posterior limb of the internal capsule and then the middle section of the cerebral peduncle - They enter the basal pons and continue as the pyramids in the medulla - At the decussation of the pyramids, at the junction between between the medulla and spinal cord, the lateral corticospinal tract crosses and then continues down the spinal cord

Describe the pathway of the ALS tract and what it trasmits.

- Pain, temperature - Synapse in the dorsal spinal cord and decussate at anterior white commisure IN THE spinal cord - Travel up, synapse in the VPL and VPN of thalamus - Go to primary somatosensory cortex, cingulate cortex, and insular cortex

People with REM sleep behavior disorder commonly also have what other disorder?

- Parkinson's disorder

Porphyria cutenea tarda vs acute intermittent porphyria

- Porphyria cutanea tarda: insufficiency of uroporphyrinogen decarboxylase; porphyria + skin findings - Acute intermittent porphyria: insufficiency of porphobilinogen deaminase; neuro + psych + abdominal pain

What can happen to muscles after prolonged hypoxia?

- Post-hypoxic Myoclonus - Myoclonus status epilepticus - acute form of PHM that typically occurs within the first 24 hours of the hypoxic insult - generalized, often symmetric, myoclonus that involves the axial, limb and facial muscles - persistent MSE is considered a marker of poor prognosis - There is also a chronic form (Lance-adams syndrome) which presents days to weeks after the insult, typically once the patient has regained consciousness

Something sitting on what artery is likely to compress the oculomotor nerve?

- Posterior communicating artery

Treatment for Bell's palsy?

- Prednisone

Explain intranuclear opthalmoplegia.

- Presentation: left gaze - left eye abducts w/ horizontal nystagmus, right eye stays stationary; right gaze - right eye abducts w/ horizontal nystagmus, left eye stays stationary - Pathophysiology: demyelination of the medial longitudinal fasciculus - Characteristic finding in multiple sclerosis

What does optic nerve pallor indicate?

- Previous injury to the optic nerve either by infarction (ischemic optic neuropathy) or inflammation of the optic nerve (MS, optic neuritis)

Patient with HIV has a ring-enhancing lesion on MRI and a positive PCR for Epstein-Barr virus. What is this?

- Primary CNS lymphoma - Ring-enhancing lesions can also be Toxo in HIV patients, but wouldn't have positive Epstein-Barr

What is Canavan disease?

- Produces developmental regression at about 6 months of age - Infant develops extensor posturing and rigidity; myoclonic seizures may develop - Defect in N-acetylaspartic acid metabolism - Changes in brain white matter are widespread and may result in a spongiform appearance - causes increase in brain volume and weight aka macrocephaly

Describe Argyll Robertson pupil.

- Reacts poorly to light but accomodates well

What is ciguatoxin?

- Reef fish - Acts on voltage-gated sodium channels, leading to increased permeability to sodium and increased excitability - Abdominal discomfort, nausea, vomiting and diarrhea, followed by paresthesias, headache, fatigue, ataxia and myalgias - Peculiar sensory phenomenon of temperature reversal, specifically characterized by a tendency for cold objects to feel hot

In patients with high-grade carotid stenosis (70-99%) what should be done?

- Referral for carotid endarectomy to reduce risk of stroke

Ataxia telangectiasias: cause and findings

- Repair of double-stranded DNA breaks, ATM gene - 3 A's: ataxia, spider angiomas, IgA deficiency

65 year old man, sudden painless loss of vision. "Like a shade dropping over my eye." No floaters of flashing lights. Has hypertension and hyperlipidemia. What is most likely?

- Retinal artery occlusion - cherry red spot on imaging - Detached retina will have light flashes and floaters and a "curtain" over their visual field that will usually come from the periphery instead

Tuberous sclerosis: cause and findings

- TSC1 or 2 gene, hamartin or tubulin protein respectively - Intellectual disability - Infantile seizures or spasms - Ash leaf spots, Shagreen spots, adenoma sebaceum - Hamartoma, cardiac rhabdomyoma, renal cysts or renal angiomyolipoma

A pure sensory stroke is most likely associated with damage to what area of the brain?

- Thalamus

Most common site for ulnar nerve entrapment?

- The elbow

What does it mean to say cerebral palsy is a static encephalopathy?

- The lesion to the brain is static, it doesn't progress - HOWEVER, the deficits associated with CP may evolve as the child matures

What organism is the most common cause of brain abscesses in patients with HIV?

- Toxoplasmosis

Which two nerves are likely to be affected by herpes zoster opthalmicus?

- Travels by the trigeminal nerve - The fourth nerve can also be involved because it shares its nerve sheath with the opthalmic division of the trigeminal nerve

Trigeminal neuralgia vs atypical facial pain?

- Trigeminal neuralgia: paroxysmal, lacinating pain, unilateral - Atypical facial pain: constant, deep pain; often bilateral, but can be unilateral

Clinical presentation of carotid artery dissection?

- Unilateral head and neck pain - Ipsilateral partial Horner syndrome (ptosis and misosi without anhidrosis) - Signs of cerebral ischemia (ex. focal weakness)

Classic features of Bell's palsy?

- Unilateral mouth drooping, disappearance of the nasolabial fold, involvement of the face (weakness in closing the eye or raising the eyebrow)

A messed-up pronator drift test is a marker of what? what about Romberg?

- Upper motor neuron or pyramidal tract lesions - Romberg: impaired propioception

Upper vs lower motor neuron signs?

- Upper: spastic paresis, no atrophy (wasting), hyperreflexia, positive Babinski, no fasciculations - Lower: flaccid paralysis, severe atrophy (wasting), hypo/absent reflexes, no Babinski, fasciculations

What is neurocardiogenic syncope?

- Vasovagal syncope that occurs in response to certain triggers such as extreme heat, sight of blood, intense emotion, standing for long periods of time, or dehydration

Patients with which vitamin deficiency are at a higher risk of developing MS?

- Vitamin D deficiency

A 4 year old boy develops progressive gait ataxia and limb weakness over the course of 3 months. Neurological assessment reveals diffusely absent deep tendon reflexes, proximal muscle weakness, ophthalmoparesis, and poor pain perception in his feet. Blood tests reveal elevated creatine phosphokinase levels and abnormally high serum bilirubin levels. Further investigations of hepatic function reveal that the child has a cholestatic hepatobilliary disorder, but there is no evidence of hepatic dysfunction sufficient to cause an encephalopathy. What vitamin deficiency is this?

- Vitamin E deficiency - Vitamin E deficiency that causes neurological disease is rare, but when it dose develop it is usually during early childhood - spinocerebellar degeneration, polyneuropathy and pigmentary retinopathy

Describe intranuclear opthalmoplegia and what it suggests injury to.

- When an attempt is made to gaze contralaterally the affected eye adducts minimally, the conlateral eye abducts, however with nystagmus - Ex. Right INO - looking right, not a problem; looking left, the left eye abducts with nystagmus, the right eye is not able to adduct - Mesencephalic or pontine injury

When do you treat bacterial endocarditis surgically?

- When there is significant valvular dysfunction resulting in heart failure, if infection is presistent or difficulty to treat medically, or if septic embolization is recurrent

What are the white blood cell and protein findings in Guillain-Barre?

- White blood cells: 0-5 (normal) - Protein: 45-1,000 (high) - Cytoalbumenic dissociation

Lumbar spinal stenosis better and worse with what movements?

- Worse with extension: standing and downhill walking - Better with flexion: sitting and uphill walking

What is adrenal leukodystrophy?

- X-linked disorder that is a problem with an ATP-binding transporter in the peroxisomal system responsible for long-chain fatty acid metabolism - Long-chain fatty acids acumulate in adrenal cortical and other cells - Produced rapidly evolving brain damage in male infants or boys, with survival from symptom onset usually limited to 3 years

What should be included in the initial workup or a first-time seizure?

- serum electrolyte, glucose, calcium, magnesium, CBC, renal and liver function tests and a TOX screen

Man with history of malignancy comes in with signs of spinal cord metastases and spinal cord compression. Other than dexamethasone, what else can you do for treatment?

- steroids + surgery or radiation - Often radiation is more practical as surgery has a high morbidity rate

Patient has an aortic dissection. Soon after, he develops urinary retention, lower extremity paresis, crude touch/pain sensation loss (but vibration in tact) and diminished reflexes. What is this?

- thoracic spinal cord ischemia - Rare complication of aortic dissection due to interruption of the intercostal and/or lumbar arteries that feed the anterior spinal cord

What is the preferred first-line drug for fibromyalgia?

- tricyclic antidepressants

Describe central cord syndrome

- typically occurs with hyperextension in patients with pre-exisiting degenerative changes in the cervical spine - Damage to the central portions of the corticospinal tracts and the decussating fibers of the lateral spinothalamic tract - Characterized by weakness that is more pronounced in the upper than lower extremities

ALS causes degeneration of what parts of the brain/spinal cord?

1) Anterior horn cells (LMN) in the spinal cord and motor nuclei of the brainstem 2) Large motor neurons (UMN) in the frontal cortex 3) Corticospinal tract

For each type of myopathy, explain the clinical features, ESR, and CK levels: 1) Glucocorticoid-induced myopathy 2) Polymyalgia rheumatica 3) Inflammatory myopathies 4) Statin-induced myopathy 5) Hypothyroid myopathy

1) Glucocorticoid-induced: after steroid treatment, progressive proximal weakness and atrophy without pain or tenderness; ESR and CK are both normal 2) Polymyalgia rheu: muscle pain and stiffness in the shoulder and pelvic girdle, responds rapidly to glucocorticoids; ESR elevated, CK normal 3) Inflammatory: muscle pain, tenderness, and proximal muscle weakness; skin rash and inflammatory arthritis may be present; ESR and CK both elevated 4) Statin-induced: prominent muscle pain/tenderness without or without weakness; normal ESR, elevated CK 5) Hypothyroid: muscle pain, cramps and weakness involving the proximal muscles; features of hypothyroidism present; normal ESR, elevated CK

Clinical presentation of each of these lesions: 1) Posterior limb of internal capsule (lacunar infarct) 2) Middle cerebral artery occlusion 3) Anterior cerebral artery occlusion

1) Lacunar infarct: unilateral motor impairment, no sensory of cortical deficits, no visual field abnormalities 2) MCA occlusion: contralateral somatosensory and motor deficits (face, arm and leg), eye deviation toward side of infarct, homonymous hemianopia, aphasia, hemineglect 3) ACA occlusion: contralateral somatosensory and motor deficits, predominantly in lower extremity; emotional disturbances, urinary incontinence

Explain all the major aphasias by walking through the algorithm

1) Specify non-fluent vs fluent 2) Nonfluent: poor reptition vs good reptition - Nonfluent, poor repetition, poor comprehension: global aphasia - Nonfluent, poor repetition, good comprehension: broca's aphasia - Nonfluent, good repetition, poor comprehension: mixed transcortical aphasia - Nonfluent, good repetition, good comprehension: transcortical motor aphasia 3) Fluent: poor repetition vs good repetition - Fluent, poor repetition, poor comprehension: wernicke's aphasia - Fluent, poor repetition, good comprehension: conduction aphasia - Fluent, good repetition, poor comprehension: transcortical sensory aphasia - Fluent, good repetition, good comprehension: anomia aphasia

A 52 year old woman with diabetes awakens wtih right body weakness. Examination at the hospital indicates relatively symmetric upper motor neuron pattern of weakness involving the face, arms and legs. There are no sensory abnormalities. Language is preserved. A stroke associated wtih this presentation is most likely with damage to the following? A. Internal capsule B. Cerebellum C. Putamen D. Caudate E. Amygdala

A. Internal capsule - Pure motor stroke

A 36 year old woman comes to the office due to headaches. Over the past 3 months the patient has had constant, bilateral headaches almost every day shortly after awakening. She has had intermittent episodes of migraines since adolescence, but her current headaches are more frequent though milder. The patient has been taking over-the-counter analgesics several times a day. She has had no weakness, numbness, or vision changes. The patient has no other medical conditions and does not use tobacco, alcohol or illicit drugs. Vital signs are within normal limits. BMI is 24. Physical examination including funduscopy and neuro exam shows no abnormalities. Blood cells counts and serum chemistry studies are within normal limits. Which of the following is the most appropriate next step in management of this patient? A. Advise to stop taking analgesics B. Measure CSF opening pressure C. OBtain MRI of the brain D. Prescribe low-potency opioids E. refer for psychiatric evaluation

A. Advise to stop taking analgesics - Medication over-use headache

A 28 year old man with a history of spinal cord injury comes to the emergency department due to headache, restlessness, and excessive sweating for the past several hours. A year ago, he was involved in a motor vehicle collision that caused a C6 spinal cord injury leading to paraplegia and neurogenic bladder. He has no other medical issues. Temperature is 96.8, blood pressure is 170/110, and pulse is 52. Physical examination shows facial flushing and excessive perspiration from his face and neck. The lungs are clear on auscultation and heart sounds are normal. The abdomen is soft with a mass palpable in the lower abdomen. A suprapubic catheter is present. Neurological examination demonstrates absence of pain and temperature sensation below the nipples and paraplegic and spastic lower extremities, unchanged from previous examiantion. Further evaluation reveals occlusion of the urinary catheter and urinary retention. The urinary catheter is changed, which leads to rapid improvement in his symptos and hypertension. Which of the following best explains the observed findings in this patient? A. Autonomic dysfunction B. Carotid sinus hypersensitivity C. Catecholamine secreting tumor D. Peripheral autonomic neuropathy E. Renovascular hypertension F. Spinal shock state

A. Autonomic dysreflexia - Potentially life-threatening complication of spinal cord injury about T6 - In an intact spinal cord: sympathetic activity is modulated by higher-level neurons - Howevere, SCI results in loss of modulatory activity below the lesion - Noxious stimuli (ex. urinary retention, constipation, pressure ulcers) can precipitate an unregulated sympathetic response, leading to vasoconstriction and severe hypertension - Above the lesion, a compensatory parasympathetic response causes diaphoresis, flushing, bradycardia, and nasal congestion

A 65 year old man comes to the offie due to unsteady gait and frequent falls over the past 2 months. He left side feels "weak" which makes it difficult to maintain balance when walking or standing. He has also had headaches and nausea, which are worse in the morning before he has had coffee. The patient has a history of hypertension and type 2 DM, and he had a MI 5 years ago. He takes insulin for DM but does not routinely monitor his blood glucose. The patient is a former smoker and does not use alcohol or drugs. Blood pressure is 130/80, and pulse is 76. Ambulation results in the patient swaying to the left and using the wall to brace himself. There is increased resistance to passive flexion of the left upper and lower extremity. Which of the following is the most likely cause of his current condition? A. Brain tumor B. Diabetic polyneuropathy C. Hemiplegic migraine D. Lacunar infarction E. Neuroglycopenia F. Normal pressure hydrocephalus G. Parkinson disease H. Tabes dorsalis I. Vitamin B12 deficiency

A. Brain tumor - N/V + focal neurologic manifestations + symptoms worsening during the night

Which of the following types of visual field cuts is most often seen in patients with vitamin B12 deficiency? A. Centrocecal scotoma B. Homonymous hemianopsia C. Bitemporal hemianopsia D. Binasal hemianopsia E. Hemianopsia with central sparing

A. Centrocecal scotoma - The normal blind spot that people has extends out into the central vision in patients with this deficiency

A 46 year old woman comes to the office due to worsening lower extremity tingling and numbness. She has also had difficulty walking, especially at night. ROS is positive for hair loss. The patient had gastric bypass surgery 5 years ago and has los 66 lb. She was originally prescribed multivitamin and mineral supplementation but now only takes zinc. Physical examination shows gait atazia with loss of vibration and position sense in her feet. There is mild lower extremity edema and scattered areas of skin depigmentation. Lab studies shows hypochromic microcytic anemia. Deficiency of which of the following is the most likely cause? A. Copper B. Thiamine C. selenium D. Folate E. vitamin C

A. Copper - Brittle hair, skin depigmentation, neurological dysfunction (ataxia, peripheral neuropathy), anemia, osteoporosis - Acquired copper deficiency is most common from malabsorption from prior gastric surgery

A 60 year old right-handed man underwent heart transplantation 2 weeks ago for severe ischemic cardiomyopathy. He had an uneventful postoperative course and went home after 1 week. He is now readmitted from an outside hospital where he was admitted with headaches, increasing confusion, and a generalized seizure. He relates that he has had difficulty seeing for several days. On examination, he has a blood pressure os 180/100. His pupils are equal and reactive, but he has difficulty reading and finding objects presented to him. Motor and sensory functions are normal. An MRI shows several areas of T2 signal abnormality in the occipital and parietal lobe white matter bilaterally. A diffusion-weighted MRI sequence, sensitive to the changes of acute infarction, is negative. Most consistent with what? A. Cyclosporine toxicity B. Steroid psychosis C. Occipital lobe infarction D. Ischmic optic neuropathies E. Retinal detachment

A. Cyclosporine toxicity - Cyclosporine and tacrolimus may induce a syndrome resembling hypertensive encephlopathy - Headache + visual dysfunction + confusion + seizures - Usually accompanied by hypertension

A 36 year old woman comes to the emergency department due to severe headache and vision disturbance. The symptoms began abruptly an hour ago, and since then she has also felt weak, dizzy, and nauseated. The patient has not other similar symptoms in the past. Medical history is significant for a prolactinoma diagnosed a month ago, which is being treated with cabergoline. She does not use tobacco, alcohol or illicit drugs. temp is 99.7, supine blood pressure is 80/50 with a decrease to 60/40 on standing and pulse is 110. On physical examination, the patient appears anxious and in moderate distress. There is mild ptosis and impaired adduction on the right. Bitemporal hemianopsia is present. Bilateral upper and lower extremity muscle strength are normal. The neck veins are flat, heart and lung sounds are normal, and there is abdominal tenderness or extremity edema. Which of the following is the most likely underlying cause of this patient's hemodynamic change? A. Decreased adrenal hormone levels B. Dopaminergic medication side effect C. Increased global intracranial pressure D. Loss of central sympathetic output E. subarachnoid blood induced vasospasm

A. Decreased adrenal hormone levels - Patient has a pituitary adenoma (prolactinoma) - it tells you this - She most likely now has pituitary apoplexy - bleeding into the tight anntomic space of the sella turcica can cause sudden-onset, severe headache, as well as visual disturbances and eye dysfunction due to compression of the oculomotor nerve or optic chiasm - Additionally, loss of all pituitary function, in the acute setting of loss of ACTH - adrenal crisis

A 65 year old man was diagnosed with lung cancer 6 months ago. Over the past 2 months, he has had worsening severe proximal muscle weakness. He is most likely to have which of the following? A. Dermatomyositis B. Trichinosis C. Multiple sclerosis D. PML E. MG

A. Dermatomyositis - Dermatomyositis occurs as a paraneoplastic syndrome in about 15% of cases overall - Can hpappen with cancers of the GI tract, breasts, ovaries, lung, or other organs

A 29 year old woman presents wtih weakness in several muscles in different limbs. The pattern is lower motor neuron and does not fit with any particular peripheral, plexus, or root localization. Which of the following is the most common cause of mononeuropathy multiplex? A. Diabetes mellitus B. Temporal arteritis C. Sarcoidosis D. Systemic lupus erythematous E. Periarteritis nodosa

A. Diabetes mellitus

A 65 year old woman comes to the ED for transient weakness. She experienced muscle weakness in her right arm and leg this morning that lasted approximately 20 minutes and has since resolved completely. The patient has had no headahce, vision loss, or vomiting. She has never before had similar symptoms. Medical history is significant for hypertension and diet-controlled type 2 diabetes. The patient underwent mitral valve replacement with a bioprostethic valve 5 years ago due to mitral valve endocarditis. She is a life-long nonsmoker. Blood pressure is 131/70 and pulse is 65 and regular. Chest examination is unremarkable. Neurologic examination reveals no muscle weakness or sensory loss. ECG, complete blood count, and renal function are within normal limits. In addition to a neuroimaging study, which of the following is the best next step in management of this patient? A. Echo B. EEG C. Hypercoagulability studies D. Peripheral blood smear E. Serum antinuclear antibodies

A. Echo - This was a TIA and in her case, given her prosthetic valve, you should be thinking of embolization in the setting of prosthetic valve thrombosis (PVT) - You need to evaluate the valve and the thrombus with an echo - Hypercoagulability studies are really only indicated if the initial workup fails to determine an etiology

A 34 year old man develops progressive depression and memory impairment over the past 6 months. His initial neurological evaluation reveals a metabolic acidosis associated with his dementia. His liver is firm and his spleen appears to be slightly enlarged. He has tremor and rigidity in his arms and walks with relatively little swing in his arms. His blink is substantially reduced, which gives him the appearance of staring. An MRI of the brain reveals some atrophy of the putamen and globus pallidus. His CSF is normal. His EEG is unremarkable. What is the most likely? a. Hepatolenticular degeneration B. Hyperparathyroidism C. Central pontine myelinosis D. Akinetic mutism E. MPTP poisoning F.Locked-in syndrome G. Postencephalitic parkinsonism H. Neuroleptic effect I. Essential tremor J. Vegetative state K. Hypermagnesemia

A. Hepatolenticular degeneration - AKA Wilson disease!

Person exposed to very loud noises. Hearing loss best described by what? A. High-tone sensorineural loss B. Low-tone sensorineural loss C. High-tone conductive loss D. Low-tone conductive loss E. Central deafness

A. High-tone sensorineural loss - The principal site of damage with acoustic trauma is the cochlea

An obese 60 year old woman with diabetes comes to the office complaining of "balance problems when walking." She has had two falls over teh last two months. She also has tingling in her feet and dull pains in her back and legs. The neurological examination reveals decreased muscle strength in both legs, diminshed propioception peripherally in both feet, decreased pain sensation in her feet and ankles, a hyperactive right knee jerk, and an upgoing plantar relflex on the right side. Her last HbA1c was 7.6%. Whta is the next best step in managing this patient's condition? A. MRI of the spine B. Nerve conduction studies C. Physical therapy referral D. Repetitive stimulation electromyography E. Tight blood glucose control

A. MRI of the spine - Hyperactive reflexes and an upgoing plantar relfex do not fit with diabetic neuropathy and you have to evaluate for an upper motor neuron process

A 17 year old girl presents with subacute mental status change and left arm weakness. She had a viral illness 1 week ago, and now a diagnosis of acute disseminated encephalomyelitis is made. ADEM is a white matter disease that is distinguishable from MS by which of the following? A. It's monophasic B. Rapidly lethal C. Associated with brainstem and spinal cord disease D. Associated with MRI lesions, which may resolve E. Associated with inflammatory changes in the brain

A. Monophasic - Indistinguishable from MS on MRI - It only occurs on a single occasion, and not in a recurrent pattern like MS - usually develops within days to weeks of a viral illness or vaccination - As in MS, the lesions associated with ADEM usually produce demyelination with sparing of the nerve axons

A 24 year old man comes to the ED for evaluation of eye pain and swelling. He was involved in an altercation last night after significant alcohol use but does not remember any further details. The patient has double vision when he attempts to look up. On examination, the left side of the face is signficantly swollen, tender, and ecchymotic. visual acuity is normal. He is unable to look up with his left eye. Which of the following is the most likely reason for this patient's diplopia? A. Muscle entrapment B. Open globe entrapment C. Orbital cellulitis D. Orbital hematoma E. Trochlear nerve injury

A. Muscle entrapment injury - Orbital floor fracture common with face trauma in a fight - "Blowout fracture: - herniation of orbital fat into the maxillary sinus and entrapment of the inferior rectus muscle

A 7 year old girl acutely develops horizontal diplopia that worsens over the course of a few days. Examination reveals that the double vision is exacerbated by leftward gaze. REd glass testing reveals that the "false" image is coming from the left eye. She is most likely to have which of the following? A. Pontine glioma B. Medullary glioma C. Mesencephalic infarction D. Pontine infarction E. Medullary infarction

A. Pontine glioma - Another lateral rectus palsy - So we know it is the pons - Glioma will grow and damage the nerve

A 37 year old man has difficulty relaxing his grip on his golf club after putting. He also is excessively somnolent. Examination reveals early cataract development, testicular atrophy and baldness. His family says that he has become increasingly stubborn and hostile over the past 3 years. His ECG reveals a minor conduction defect. An EMG will probably reveal which of the following? A. Repetitive discharges with minor stimulation B. Polyphasic giant action potentials C. Fasciculations D. Fibrillations E. Positive waves

A. Repetitive discharges with minor stimulations - Myotonic dystrophy - This EMG pattern is often referred to as a "dive bomber" pattern because the characteristic sound produced when the evoked action potentials are heard

Which of the following retinal pathologies tend to occur in people with tuberous sclerosis? A. Retinal phakomas B. Retinitis pigmentosa C. Retinal telangiectasias D. Retinoblastomas E. Retinal problems are not part of the disease

A. Retinal phakomas - Gliomatous tumors that require no treatment - Other findings for TS: ash-leaf spots, shagreen patches, CNS calcifcations ,renal tumors, cardiac rhabdomyomas, and epilepsy

A 2 year old male child has recently been diagnosed wtih muscular dystrophy. The parents are highly educated people, but not in the medical field. They have many specific and detailed questions. For a female child to have Duchenne dystrophy, she must have which of the following? A. Turner syndrome B. Klinefelter syndrome C. Two affected parents D. An affected father E. An affected brother

A. Turner Syndrome - To get Duchenne, you need to have all messed up 2 messed up X's - With Turner's they only have the 1 x, so if it is messed up they can get it

A 30 year old man is brought to the ED due to worsening headache. The patient has had episodic right-sided headache over the past 6 months. Thirty minutes ago, he began experiencing a right temporal headache while at rest, which gradually worsened to severe pain over the next several minutes. He also had nausea and an episode of vomiting. The patient is now somnolent and difficult to arouse. He has no other medical conditions and had a normal medical evaluation a year ago for military enlistment. Temp is 98.6, blood pressure 150/90, pulse is 64, respirations are 14. The patient withdraws all extremities to painful stimuli, but left-sided deep tendon reflexes are increased. There is no neck rigidity. Which of the following is the most likely underlying cause of this patient's current condition? A. Arteriovenous malformation B. Carotid artery atheroma C. Cerebral amyloid angiopathy D. Dural venous sinus thrombosis E. Hypertensive vasculopathy F. Paroxysmal cardiac arrhythmia

A. arteriovenous malformation - Headache + nausea + vomiting + decreased consciousness - suggestive of intracranial bleed - Patient's young age and history of right-sided headaches suggests a rupture AVM - AVMs occur when an artery directly anastamose with the veins without a capillary bed in between - Patients typically diagnosed before age 40, most commonly after developing an IVH

A patient with meningitis and facial weakness of unknown etiology has been given isoniazid and rifampin. There was no improvement and she is treated wtih high-dose steroids. Within 1 week of the introduction of prednisone, she develops pain radiating down the back of ther right leg and has difficulty dorsiflexing the right foot. This new symptom most likely represents which of the following disorders? A. Borrelia radiculopathy B. Diabetic mononeuritis multiplex C. Isoniazid neuropathy D. Rifampin toxicity E. Tuberculous radiculopathy

A. borrelia radiculopathy - Remember, she had a facial weakness of unknown etiology

A girl has Hartnup disease. Her condition may respond to large supplementary doses of which of the following? A. Vitamin C B. Nicotinamide C. Thiamine D. Pyridoxine E. alpha-Tocopherol

B. Nicotinamide - Hartnup: intestinal malabsorption of tryptophan and other neutral amino acids - Tryptophan is the precursor for nicotinamide, so you can supplement with nicotinamide

A 54 year old woman presents with 6 months of progressive memory loss. she has limited verticla eye movements and on examination, she has rhythmic, synchronous grimacing and eye closure movements. Jejunal biopsy reveals PAS-positive cells. What is the most likely? A. HTLV-1 B. Tropheryma whippeli C. Treponema pallidum D. JC virus E. Prior protein F. CMV G. Herpes simplex virus H. Taenia solium

B. - Whipple disease: rare multisystem disorder - GI complaints such as abdominal pain, weight loss - CNS infection cause causes seizures, myoclonus, ataxia, supranuclear gaze disturbances, hypothalamic dysfunction and dementia - You can have CNS problems without GI disease - Diagnosis is made with biopsy of the jejenum - PAS positive cells - Tx. antibiotics

With agenesis of the corpus callosum, magnetic resonance imaging (MRI) will reveal which of the following? A. Atrophy of the frontal lobes B. Abrnomally shaped lateral and third ventricles C. Cerebellar ataxia D. Schizencephaly E. Encephaloclastic porencephaly

B. Abrnormally shaped lateral and third ventricles - Lateral ventricles develop a typical batwing conformation - Patients with this may be asymptomatic, but also may hace a variety of cognitive disorders

A 91 year old woman has 3 days of gradually worsening fever and headache. She then develops blurry vision and a stiff neck. Her granddaughter becomes concerned and brings her to the ED. MRI with contrast has an enhancement pattern suggesting rhomboencephalitis. CSF shows a mild pleocytosis with no organisms. All blood and CSF cultures are negative. Which of the following medications is the best treatment for the organism likely responsible? A. Penicillin G B. Ampicillin plus gentamicin C. Tetracycline D. Ceftriaxone E. Rifampin

B. Ampicillin plus gentamicin - Based on her age, this is likely to be Listeria

A 50 year old is being evaluated for tinnitus. It is worse on some days than others. Which of the following shoudl he be told may exacerbate the tinnitus? A. Alcohol B. Aspirin C. Glucose D. Diazepam E. Steroids

B. Aspirin - asprin may produce tinnintus in people usually unaffected by this problem - Patients on high-dose aspirin for rheumatoid arthritis are especially susceptible to this drug-induced tinnitus

A 22 year old male heroin user has been having severe headaches during sexual intercourse. Within a few minutes of one headache, he develops right-sided weakness, and become stuporous. His neurologic examination reveals neck stiffness as well as right arm and face weakness. An unenhanced emergency CT scan reveals 3-4 cm lesion in the cortex of the left parietal lobe. The addition of contrast enhancement reveals two other smaller lesions in the right frontal lobe but does not alter the appearance of the lesion in the left parietal lobe. Which of the following diagnostic studies is most likely to establish the basis for this patient's neurologic deficits? A. HIV antibody testing B. CSF examination C. EEG D. Nerve conduction studies E. Cardiac catheterization

B. CSF examination - This is most likely bleeding from a mycotic aneurysm (infected aneurysm) - Mycotic aneurysms form over the cerebral convexities with subacute bacterial endocarditis; aneurysm develops from an infected embolus originating on the diseased heart valves and lodging in the arterial wall - Aneurysms are especially likely to bleed during exertion - The fact that the lesion appeared the same on unenhanced and enhanced CT scans suggests it is a hematoma - CSF for this would show yellow, with many red blood cells or grossly bloodly, therby providing evidence of a recent subarachnoid hemorrhage

A 30 year old man comes to the office with fatigue and lethargy that has worsened over the last 2 weeks. He has been forgetful lately and feels "exhuasted" at the end of teh day. The patient works as a contractor and is currently renovating old houses for sale. He describes feeling "clumsy" and dropping things at work, as well as tripping multiple times while climbing stairs. He also has abdominal pain that he attributes to constipation. The patient drinks 1-2 beers each weekend and does not use tobacco/drugs. His mother was diagnosed with lupus and his older sister had thyroid surgery. Blood pressure is 120/80, pulse is 76. Examination shows normal jugular venous pressured, no thyromegaly, clear lung fields, and normal first and second heart sounds. The abdomen is soft and nontender. There is no hepatomegaly or splenomegaly. There is weakness on dorsiflexion of bilateral wrists and feet. Upper and lower limb deep tendon reflexes are 1+. Creatinine phosphokinase levels are normal. Which of the following is most likely to improve this patient's symptoms? A. Allopurinol B. Calcium disodium EDTA C. Cobalamin supplementation D. Levothyroxine E. Plasmapharesis F. Thiamine supplementation

B. Calcium disodium EDTA - This is lead poisoning

Highest risk for which of these with prolonged status epilepticus? A. Cerebellar atrophy B. Cortical necrosis C. Obstructive hydrocephalus D. Parkinson's disease E. Pontine hemorrhage

B. Cortical necrosis - Hallmark of prolonged seizures and can lead to persistent neurological deficits and recurrent seizures

A 65 year old woman comes to the offie for evaluation of progressively worsening memory. The patient lives alone and considers herself "very independent"; however, she has recently forgotten to pay her bills on several occasions and also lost her way while driving in a previously familiar neighborhood 2 weeks ago. ROS reveals no other symptoms except for mild urinary urgency. The patient has a history of hypertension and hyperlipidemia. She does not use alcohol, tobacco, or drugs. Temp is 98, blood pressure is 130/90, pulse is 72 and respirations are 14. The lungs are clear to auscultation, and a grade 2/6 systolic ejection murmur is present. Abdominal examination shows no tenderness or masses. The cranial nerves are intact, funduscopy is normal, and there is no tremor. The patient immediately recalls all 3 of 3 items read to her, but 5 minutes later she cannnot remember any of them. her gait is broad-based and shuffling. Blood cell count and serum chemistry studies are within normal limits. Which of the following is the likely underlying cause of this patient's symptoms? A. Alpha-synuclein dysfunction is dopaminergic neurons B. Decreased CSF absorption C. Excessive cerebral amyloid peptide deposition D. Neurocognitive effects of B12 deficiency E. Selective loss of frontotemporal neurons

B. Decreased CSF absorption - This is normal-pressure hydrocephalus

A 20 year old ataxic woman with a family history of Friedreich disease develops polyuria and excessive thirst over the course of a few weeks. She notices that she becomes fatigued easily and has intermittently blurred vision. Which of the following is the most likely explanation for her symptoms? A. Inappropriate ADH B. Diabetes mellitus C. Panhypopituitarism D. Progressive adrenal insufficiency E. Hypothyroidism

B. Diabetes mellitus - More than 10% of patients with Friedreich develop diabetes mellitus - Visual problems occur with the hyperglycemia of uncontrolled diabetes, but even Friedreich patients without diabetes develop optic atrophy late in the course of the degerative disease

An 18 year old woman comes to the office due to recurrent headaches. In the past, the patient has had occasional headaches, espeically during term exams or other stressful situations. However, over the past 3 months she has had 4 episodes of severe headache. They are always left-sided, throbbing, associated wtih nausea and spontaneously resolve after 12-24 hours. The patient also reports feelings of numbness and tinlging in her extremities prior to the headache. She has no chronic medical conditions and underwent an appendectomy several years ago. The patient is sexually active with her boyfriend and began taking oral contraceptives 6 months ago. Vital signs are normal. PE, including neurological examination is also normal. Which of the following is the next best step in management of this patient? A. Begin SSRIs B. Discontinue oral contraceptives C. Measure CSF pressure D. Order MRI of the brain E. Prescribe daily propranolol F. Reassure and advise NSAIDs as needed

B. Discontinue oral contraceptives - she has migraines with aura - contraindication to OCPs

A 75 year old man comes to the offie due to intermittent right eye vision loss. The patient reports 3 episodes over the past 2 months in which he suddenly experienced a "curtain falling over the right eye" for several minutes before it spontaneously resolved. He has no eye pain or discomfort, focal weakness, numbness, or headache. His medical history if significant for hypertension, hyperlipidemia, and osteoarthritis. He smoked for 25 years before quitting at age 50. Blood pressure is 140/85, pulse is 74. Visual acuity, pupillary reflex, and funduscopic examinations are unremarkable. Further neurologic examination shows normal reflexes, motor strength, and sensation. Which of the following is most likely to yield the diagnosis in this patient? A. CT scan of the head B. Duplex U/S of the neck C. Echo D. ESR E. LP and CSF analysis F. MRI of the brain G. Ocular tonometry

B. Duplex U/S of the neck - This is amaurosis fugax - painless, rapid, and transient vision loss (< 10 minutes) - Most common etiology is retinal ischmia from atherosclerotic emobli originating from the ipsilateral carotid artery

A 35 year old man is brought to the ED due to weakness and lethargy. His friend could not get him out of bed this morning and called EMS. The patient has a history of injection heroin use, but IM naloxone administered by EMD personall failed to improve his condition. Temp is 100.6, blood pressure is 110/66, pulse is 104. and respirations are 19. There is ptosis of both eyes and the pupils are dilated and sluggish to react. The patient is unable to hold his head up and respirations are shallow. The lungs are clear on auscultation and no cardiac murmurs are present. The abdomen is soft and nontender with normal bowel sounds. Two abscesses are present on the left thigh. Arterial blood gases reveal hypoxia and acute respiratory acidosis. The patient is intubated and mechnical ventilation is begun. Whcih of the following is the next best step in management of this patient? A. Comprehensive drug screen B. Equine botulinum antitoxin C. IV buprenorphine D. IV naloxone infusion E. Tetanus immune globulin

B. Equine botulinum antitoxin - Wound botulism - Opioid intoxication would have been reverse by naloxone, and with other drug intoxications, ptosis is uncommon

Which of the following is the most appropriate treatment for presumed optic neuritis in a patient? A. Oral prednisone B. IV methylprednisolone C. Cyclophosphomide D. Plasma exchange E. IV gamma globulin

B. IV methylprednisolone - Clinical trials have shown the IV methylprednislone for an attack of optic neuritis is associated with better outcome than oral prednisone

A 70 year old man presents to the emergency room after experiencing a 15-minute episode of slurred speech and arm weakness. He was eating breakfast with his wife and son when he suddenly unable to articular words well. He had difficulty holding up his fork with his left ahnd and he had some left facial drooping. The episode lasted 15 minutes and resolved spontaneously. He has a history of hypertension and diabetes, and smoking. His neurological examination at the time of presentation is normal. His CT scan shows a small area of hypodensity in the right posterior limb of the internal capsule. What is the most likely diagnosis? A. TIA B. ischmic stroke C. Intracranial neoplasm D. Simple partial seizure E. Intracerebral hemorrhage

B. Ischemic stroke - Don't get confused by the time course - TIA requires that there is no evidence of acute infarction on imaging - Strokes can have resolved symptoms by the time of presentation

A 23 year old woman presents to the ER with complaints of "spells" for 3 months. Per her and her family she has a sense "that something terrible is going to happen" before she blacks out. Family notes that she will pick at her clothes, mutter and stare off and is not responsive to them for 1-2 minutes during these episodes. Afterwards she is confused for about an hour and "wiped out" for about a day. Her neurologic exam in clinic and recent MRI are normal. Which of the following is most likely to improve her symptoms? A. Aspirin B. :Lamotrigine C. Nortriptyline D. Paroxetine E. Propranolol

B. Lamotrigine - These is temporal lobe epilepsy

The olfactory cortex in humans is located in which of the following locations? A. Anterior perforated substance B. Lateral olfactory gyrus C. Posterior third of the first temporal gyrus D. Angular gyrus E. Calcarine cortex

B. Lateral olfactory gyrus

As the caudate nucleus atrophies, the shape of which of the following is affected? A. Cerebellum B. Lateral ventricle C. Third ventricle D. Lenticular nuclei E. Temporal lobe

B. Lateral ventricle - Gives it a classic boomerange or comma appearance that you see in Huntington's disease

A 79 year old woman is brushing her teeth when she has an intesnse sensation that the room is moving as if she were on a ship. Examnination and testing reveal a cerebellar stroke. Cerebellar damage may be associated with severe vertigo if the tissue is damaged is in the distribution of which of the following arteries? A. Superior cerebellar artery B. Posterior inferior cerebellar artery C. Anterior inferior cerebellar artery D. Anterior spinal artery E. Posterior cerebral artery

B. PICA - The PICA has medial and lateral branches - The medial branches supply the brainstem - With occlusion of these, the vestibular nuclei in the brainstem are infarcted, and vertigo is common

Which of the following is the most common form of retinal degeneration? A. Serous retinitis B. Retinitis pigmentosa C. Confluent drusen D. Drug-induced retinopathy E. paraneoplastic retinal degeneration

B. Retinitis pigmentosa - Hereditary degenerative disease

A patient undergoes ventriculoperitoneal shunt placement for hydrocephalus. He is discharged 2 days later, his gait and cognition much improved. The following morning, his wife finds him lying in bed, very confused and complianing of a headache. He is unable to walk. The surgeon who performed the procedure is concerned that these new symptoms are owing to which of the following? A. Chemical meningitis B. Subdural hematoma C. Epidural hematoma D. seizures E. Bacterial ventriculitis

B. Subdural hematoma - Up to 28% of patients who undergo this proceudre may suffer major complications such as subdural hematoma - The reduction in ICP brought on by the reduction in CSF volume may cause the brain to pull away from the covering meninges, stretching and potentially ruptuing the bridging veins

A 48 year old left-handed man develops increased sensitivity to sound in his left ear. A brain MRI reveals a posterior fossa mass. This symptom may develop in one ear with damage to which of the following ipsilateral CNs? A. V B. VII C. VIII D. IX E. X

B. VII - Facial nerve innervates the stapedius muscle - With paralysis of this muscle, undamped transmission of acoustic signals across the stapedius bone of the middle ear produced hyperacusis

A 7 year old child has a head CT ordered because of recent headaches. The scan is significant for a right parietal parenchymal defect that is continuous with the ventricle and does not appear to be lined with gray matter. This type of lesion usually develops as a consequence of which of the following? A. Fetal alcohol syndrome B. Vascular or other destructive injuries to the brain C. Trisomy 13 D. Trisomy 21 E. Dandy-walker syndrome

B. Vascular or other destructive injury to the brain - In utero damage to the fetal brain may be evident at birth as large cysts in the brain - The presence of one or more of these cysts is called porencephaly

A 35 year old woman falls 12 feet from a ladder and fractures her c-spine, causing damage at the C4 level. She is initially a flaccid quadriplegic with areflexia. This areflexia and flaccidity usually evolve into hyperreflexia and spasticity within which of the following time periods? A. 2 to 4 months B. 1 to 2 months C. 3 days to 3 weeks D. 1 to 3 hours E. 5 to 25 minutes

C. 3 days to 3 weeks - Spinal shock is a transient phenomenon that occurs with damage to fibers from upper motor neurons - Spasticity that evolves within a few days of the spinal cord injury is presumed to represent exaggeration of the normal stretch reflexes in the limbs disconnected from upper motor neuron control

A 72 year old woman presents with the sudden onset of double vision. The second image disappears if she covers either eye. Which of the following nerves is most likely to be impaired in this patient? A. oculomotor B. Trochlear C. Abducens D. Ciliary E. Muller

C. Abducens - A lateral rectus palsy will give you double vision

A 42 year old man describes difficulty walking and numbness of his legs. His tingling and numbness began 8 months ago at his toes, spread over severeal weeks to involve his legs, and in the last couple of weeks tingling spread to his fingers. He now has more difficulty getting up from a chair and he feels unsteady walking. Two years ago he had similar but less severe symptoms but resolved after severeal weeks and he never saw a doctor. on examination, cranial nerves and mental status are normal. He has mild weakness of distal muscles in the arms and distal and proximal muscles in his legs. Tendon reflexes are absent. His gait is unsteady and he has a postivie Romberg's sign. Vibratory perceptor is absent in both feet and reduced in his fingers. Nerve conduction studies showed marked reduced conduction velocitis. Examination of his spinal fluid showed elevated protein. What is the most likely diagnosis? A. ALS B. Charcot-Marie-Tooth C. CIDP D. Guillain-Barre syndrome E. Vitamin B12 deficiency

C. CIDP - Remember, this looks like AIDP, but longer time course >4 weeks, usually 8+ = CIDP

How can papillitis be distinguished from papilledema? A. Degree of swelling of the optic disc B. Associated homonymous hemianopsia C. Characteristic visual loss D. Associated limitation of eye movement E. Loss of red reflex

C. Characteristic visual loss - Visual loss is usually substantial with papillitis - inflammation of the optic nerve head - Visual loss inconsequential with papilledema - Papillitis often is an early sign of multiple sclerosis

A 54 year old man comes to the office due to recurrent falls. During ambulation, the patient predominantly flexes his right hip and knee and the right foot slaps to the floor with each step. Romberg sign is absent. Which of the following is the most likely cause of this gait abnormality? A. Basal ganglia lesion B. Cerebellar dysfunction C. Common peroneal neuropathy D. Corticospinal tract lesion E. Damage to the frontal lobe cortico-cortical white matter tracts F. Sensory neuropathy

C. Common peroneal neuropathy - Remember, this dorsiflexes the foot so if he can't do that, he slaps his foot when he tries to lift it up A. lesions to the basal ganglia give you a slow shuffling gait B. cerebellar ataxia is like a drunk person D. Corticospinal tract lesion - spastic ataxia E. cortico-cortical white matter tracts - seen in NPH - magnetic ataxia

Routine funduscopic examination of a 52 year old man reveal small, discrete red dots located in largest numbers in the paracentral region. Such retinal microaneurysms most often occur with which of the following? A. Sarcoidosis B. Chronic hypertension C. Diabetes mellitus D. Anterior communicating aneurysms E. Chorioretinitis

C. Diabetes mellitus - One of the first manifestations of DM - A proliferative retinopathy may occur with these microaneurysms

A 52 year old diabetic man on multiple medications develops vertigo. Which of the following may cause a toxic labryrinthitis? A. Promethazine B. Penicillin C. Dimenhydrinate D. Acetylsalicylic acid E. none of the above

D. Acetylsalicylic acid - Salicylates, as well as alcohol, quinine, and aminoglycoside antibiotics, may produce a toxic labyrinthitis with vertigo as a prominent feature

A 34 year old man comes to the ED due to difficultly swallowing and blurred vision that started last night. He also reports having mild shortness of breath that started today. The patient returned home to Connecticut yesterday from a 7-day Caribbean trip during which he snorkeled, ate locally cured fish, and hiked in a forest. He had a mild upper respiratory illness 2 weeks ago that spontaneously resolved; otherwise he has no other medical problems. The patient does not use tobacco, alcohol or illicit drugs. Temp is 98.6, blood pressure is 115/70, and pulse is 88. His oral mucosa is dry and speech is slurred. Pupils are dilated and react sluggishly to light. Neck muscles are weak and he has difficulty holding his head up. Muscle strength is 1/5 in the bilateral upper limbs and 5/5 in the lower limbs. There are no sensory abnormalities. Which of the following is the next most appropriate step in management of this patient? A. Atropine and pralidoxime B. Edrophonium test C. Equine antitoxin therapy D. IV methylprenisolone E. Skin examination for an attached tick F. Therapeutic plasma exchange

C. Equine antitoxin therapy - Botulism from the cured fish - This is the treatment

A 60 year old man comes to the office due to right-sided neck and shoulder pain. The pain worsens with movement and is associated wtih numbness in his forearm. Over teh past 2 years, he has had severeal episodes of similar symptoms that improved wtih rest and ibuprofen. The patient has no history of neck or shoulder trauma. He smokes a pack of cigarettes daily and has used injection drugs in the past. Vital signs are normal. PE shows limited neck rotation and lateral bending. There is decreased pinprick sensation on the posterior aspect of the right forearm but no muscle weakness. Triceps reflex is normal. Left upper and bilateral lower extremity examination show no other abnormalities. A plain radiograph of the cervical spine is ordered. The patient's symptoms are more likely associated with which of the following radiographic findings? A. Atlantoaxial subluxation B. Compression fracture C. Facet joint osteophytes D. Focal osteolytic lesions E. Odontoid process fracture F. Vertebral body squaring

C. Facet join osteophytes - Cervical radiculopathy due to cervical spondylosis

A 19 year old man is hit in the face with a lead pipe. The ocular motor muscle most likely to be injured in this case is that innervated by which of the following? A. Superior division of the third cranial nerve B. Inferior division of the third cranial nerve C. Fourth (trochlear) cranial nerve D. Sixth (abducens) cranial nerve E. Long ciliary nerve

C. Fourth cranial nerve - Extends far into the orbit and is vulnerable to head trauma - Third nerve is vulnerable to pressure from aneurysms, but it is usually not disturbed with head trauma

A 35 year old woman who is known to be HIV positive is found to be positive for JC virus. WHat is the most appropriate treatment? A. Amphotericin B B. Cranial radiation C. Highly active antiretroviral therapy D. IV acyclovir E. IV ceftriaxone

C. HAART - Prognosis is poor but HAART has been shown to prolong survival

A 42 year old previously healthy woman comes to the office due to worsening double vision and gait unsteadiness. She states that she had cramping abdominal pain and diarrhea 2 weeks ago after an outdoor picnic, whcih spontaneously resolved after 3 days. The double vision began 4 days ago and is persistent and progressive. The patient has also been stumbling while walking due to incoordination. She has had no fever, headahce, neck pain, photophobia, or bowel or bladder dysfunction. The patient does not use tobacco, alcohol or illicit drugs. Vital signs are within normal limits. On physical examination, she is fully alert and oriented with normal memory, speech, and language comprehension. There is mild ptosis of the right eye with weakness of the medial and upward gaze. Left eye movements are normal. Bilateral lower-extremity weakness with loss of deep tendon reflexes are present. There is dysmetria on heel-to-shin testing on both sides. Bilateral upper-extremity muscle strength, reflexes and coordination are normal. Sensation to touch and pinprick is normal throughout. The remainder of the examination shows no abnormalities. Which of the following is the most likely cause of this patient's condition? A. Botulism toxin ingestion B. Dietary thiamine deficiency C. Immune-mediated nerve injury D. Neuroinvasive virus infection E. Vertebrobasilar artery occlusion

C. Immune-mediated nerve injury - This is Miller-Fisher Syndrome, a varian of Guillain-Barre syndrome - These are immune-mediated polyneuropathies

A patient with Parkinson's disease has which of the following? A. Intranuclear inclusion bodies B. Intranuclear and intracytoplasmic inclusion bodies C. Intracytoplasmic inclusion bodies D. Neurofibrillary tangles E. Amyloid plaques

C. Intracytoplasmic inclusion bodies - These are describing Lewy bodies

A 56 year old man comes to the office for evaluation of right leg pain and numbness. The pain started 2 days ago during an 8 hour car ride during which the patient was a passenger in the backseat of a small car. Midway through the car ride, he began to have numbness and burning pain over the lateral aspect of his right thigh. The patient tried stretching to relieve the pain, but it only worsened, and he is now unable to wear a belt due to the discomfort. He has type 2 DM and has gained 25 lb over the past year. BMI is 42. On examination, lower extremity strength is 5/.5 bilaterally. The right leg has a large area of numbness over the upper lateral thigh. Straight leg raise is negative. Reflexes are symmetric and intact. Pain is reproduced on hip extension and with palpation immediately below the ASIS. Compression of which of the following nerves if the most likely cause of this patient's presentation? A. Common fibular (peroneal) nerve B. Ilioinguinal nerve C. Lateral femoral cutaneous nerve D. Obturator nerve E. Sciatic nerve

C. Lateral femoral cutaneous nerve - This is called meralgia paresthetica - Compression of the LFCN at the inguinal ligament, typically caused by tight belts or clothing or injury during surgery - Obesity and diabetes are risk factors - Strength is normal because the LFCN is purely a sensory nerve

A 34 year old man comes to the physician after several days of difficulty walking. He also describes some "funny" sensations in his feet. He has no recent skin rash, diarrhea, or joint pain. He has had lower back pain for the last week due to a lifting injury while he was moving. He visited friends in Connecticut 1 month ago and while there developed an upper respiratory infection that lasted 4 days. He smokes 1 pack of cigarettes daily and admits to occasional IV drug use. He has not been sexually active for 3 months. His temp is 98, blood pressure is 120/80, and pulse is 90. Chest exam is normal. The abdomen is soft and nontender. The liver span is 8 cm and the spleen is not palpable. Cranial nerves II-XII are intact. Muscle strength is reduced in the lower extremities but well preserved in the upper extremities. Lower extremity sensation is decreased. Mild hyperreflexia is noted bilaterally in the lower extremities. Stroking the sole of the feet elicits extension of the great toe. Which of the following is most likely to diagnose this patient's condition? A. Electromyography B. CT scan of the brain C. MRI of the spine D. Serologic tests for Borrelia E. Lumbar puncture F. Edrophonium test

C. MRI of the spine - This patient has signs of a spinal cord compression - isolated, symmetric lower extremity symptoms - He has upper motor neuron symptoms - indicates central nervous system involvement - Bilateral - so can localize to the spinal cord - Compression could be caused by an abscess, disk herniation, or malignancy - Abscess is of great concern in this IV drug user - Cord compression is an emergency and is diagnosed with an MRI of the spine

A 38 year old miner develops a shuffling gait, tremor and drooling. His speech is difficult to understand and trails off in volume until it is inaudible. He consults a physician because of easy fatigability and frequent falls. Cogwheel rigidity is evident in his arms and legs. His tremor is most evident when his limbs are at rest. What exposure? A. Lead. B. Arsenic C. Manganese D. Mercury E. CO F. Ergot G. NO

C. Manganese - Manganese inhalation by miners producers a clinical picture similar to that seen with Wilson disease - Parkinsonism is the most prominent feature

A 35 year old woman presents with slowly evolving left arm ataxia, left-sided head tilt, dysarthria, and left facial weakness. The patient denies vertigo, tinnitus, or hearing loss. MRI reveals a posterior fossa mass that lies close to the bone and enhances with contrast. Whcih of the following is the most likely explanation for this lesion? A. Cerebellar infarction B. Cerebellar hemorrhage C. Meningioma D. Schwannoma E. Astrocytoma

C. Meningioma - A stroke would evolve fast, not slow ; neoplasm more likely in this case - No involvement of CN VII - no probably not Schwannoma - Astrocytoma is unlikely to occur in the posterior fossa, especially in adults

A 38 year old woman comes to the office for evaluation of pain and tingling in her hands. She works as a telemarketer for a local sales and marketing agency and has been having difficulty holding the phone. Review of systems is positive for frequent muscle aches in her arms and legs, constipation, lethargy, and difficulty concentrating at work. The patient has no other medical problems and takes no medications. She does not use tobcco, alcohol or illicit drugs. Blood pressure is 130/80, pulse is 79 and respirations are 14. BMI is 32. On examination, there is numbness in the thumb, index, and middle fingers of both hands. Deep tendon reflexes are 1+ in all 4 extremities.The skin appears dry and doughy and both elbows. Mild edema is present in both ankles. Which of the following is most likely responsbile for this patient's hand symptoms? A. amyloid accumulation B. calcium phosphate deposition C. Mucinous infiltration D. Periventricular demyelination E. Vascular steal phenomenon

C. Mucinous infiltration - She has symptoms suggestive of hypothyroidism - She also have findings consistent with carpal tunnel syndrome - Direct infiltration of the median nerve with mucinous material

A 64 year old man presents to the emergency room with convulsive seizures. A precontrast CT of the brain reveals a hemorrhagic mass in the left frontal lobe, but there is little apparent shift of brain structures and no ventricular enlargement. Two hours after the patient's seizures have stopped, his blood pressure is still elevated at 180/100 and his pulse is slow at 50 beats per minute. Although the patient is still unconsciuos, he appears to have decreased tone on the right side of his body. The physician should request which of the following interventions? A. IV clonidine to lower the blood pressure B. Placement of a cardiac pacemaker to manage the bradyarrhythmia C. Neurosurgical consult D. Placement of a ventriculoperitoneal shunt E. IV TPA

C. Neurosurgical consult - An expanding intracranial mass will produce elevated blood pressure and a slow heart rate - CUSHING RESPONSE - Man may have a neoplasm in the brain or amyloid bleed

A 50 year old man had a brainstem stroke following a vertebral artery dissection secondary to an acute sports-related injury. This patient might be expected to develop dysphagia secondary to involvement of which of the following structures? A. Nucleus solitarius B. Nucleus and descending tract of cranial nerve V C. Nucleus ambiguus D. Lateral spinothalamic tract E. Inferior cerebellar peduncle

C. Nucleus ambiguus - Contains the motor neurons that contribute to the 9th and 10th cranial nerves

A 45 year old man has noticed that over the past 6 months that his sense of smell is not as sensitive as it used to be. On examination, he has unilateral anosmia, ipsilateral optic atrophy, and contralateral papilledema. Which of the following is the most likely diagnosis? A. Pseudotumor cerebri B. MS C. Olfactory groove meningioma D. Craniopharyngioma E. Nasopharyngeal carcinoma

C. Olfactory groove meningioma - Ipsilateral optic atrophy and contralateral papilledema in association with an intracranial tumor constitute the Foster-Kennedy syndrome - A meningioma of the olfactory groove may produce this syndrome if it extends posteriorly to involve the ipsilateral optic nerve

a 10 year old boy presents to the ED at the children's hospital with new onset problems "tripping". Exam is notable for mild to moderate right-sided foot drop, weakness with eversion of his foot on the right, and an abrasion along the lateral aspect of the right knee. While he orginially denies any trauma, when asked about the abrasion, he does remember stepping through the "ice crust on the snow" recently when playing outside. The most likely cause of this presentation is compression of which of the following locations? A. Nerve compression at the L2 root B. nerve compression at the L5 root C. Peroneal nerve at the head of the fibula bone D. Posterior tibial nerve in the calves E. Sciatic nerve as it exits the sciatic notch below the buttocks

C. Peronal nerve at the head of the fibular bone

The CSF in persons with MS will typically exhibit which of the following? A. Glucose content of less than 20% of the serum content B. Persistently elevated total protein content C. Persistently elevated IgG content D. Mononuclear cell counts of greater than 100 cells per uL E. Erythryocyte counts of greater than 10 cells per uL

C. Persistently elevated IgG content

A child with rapid downward deviation of both eyes followed by slow upward conjugate eye movements probably has which of the following? A. SSPE B. MS C. Pontine glioma D. Cervicomedullary junction ischemia E. Cerebral palsy

C. Pontine glioma - This is called ocular bobbing - Usually develops with pontine damage

A 53 year old woman was taken to the ED because of confusion. She was driving and suddnely asked her sister where she was, even though they were driving to her sister's house. She seemed confused and asked repeatedly "where are we?" On examination, vital signs and cardiac examination are normal. She is oriented to her name and year, but cannot name the exact date or hospital. her speech is fluent, but repeats questions several times dueing the interview and cannot recall any of five items presented to her 5 minutes earlier. Her memory of remote events is intact. Her neuro examination is otherwise normal. After 6 house she begins to return to her baseline and retains new information presented to her, although she does not recall anything from the previous 10 hours. An EEG is normal. Her brain MRI shows punctate foci of restricted diffusion in the bilateral hippocampi, while images of head and neck are normal. Which of the following is most appropriate? A. Clopidogrel B. Ketorolac C. reassurance D. Phenytoin E. Thiamine

C. Reassurance - This is transient global amnesia

A 26 year old recent immigrant from Brazil presents to the hospital with a subacute worsening paraparesis. The patient had worked in the lumbar industry deep in the Amazon jungle. MRI of the spinal cord is abnormal, and a biopsy reveals widespread granulomas. In the midst of one granuloma is an ovoid mass with a spine extending from one side. The pathologist interprets this as a parasitic ovum. If the pathologist is correct, which of the following is the most likely cause of the lesion? A. taenia solium B. Entamoeba histolytica C. Schistosoma mansoni D. Schistosoma japonicum E. Treponema pallidum

C. Schistosoma mansoni - S. mansoni embolizes to the lumbar spinal cord - S. japonicum and s. mansoni embolize to the CNS (s. japoncium doesn't go to the lumab spinal cord) - T. pallidum can produce a granulmoatous lesion in the spinal cord, but there is an ovum in the granuloma

A 19 year old woman with headaches and visual blurring ahd prominent bulging of both optic nerve heads with obscuration of the margins of both optic discs. Her physician is reluctant to pursue neurological studies because she is 8 months pregnant and had similar symptoms during the last month of another pregnancy. Her physical and neurologicla examinations are otherwise unrevealing. If neuroimaging studies were to be performed on this woman, they probably would reveal which of the following? A. A subfrontal meningioma B. Intraventricular blood C. Slitlike ventricles D. Transtentorial herniation E. Metastatic breast carcinoma

C. Slitlike ventricles - Most likely pseudotumor cerebri - Too much CSF, compresses ventricles

A third-nerve palsy associated with diabetes mellitus is usually characterized by which of the following? A. Poor pupillodilation B. Poor pupilloconstriction C. Sparing of pupillary function D. Inversion of the affected eye E. Upward deviation of the affected eye

C. Sparing of pupillary function - Superficial fibers to the iris are usually spared with diabetes mellitus

A 6 month old child has head lag, tongue fasciculations, and bilateral abducens palsies. MRI scan reveals a type 2 Chiari malformation. which of the following defects would this child likely have? A. A renal cyst B. Pulmonary atelectasias C. Spina bifida D. Holoprosencephaly E. A hepatic cyst

C. Spina bifida - Spina bifida may be extreme in some of the children affected by type 2 chiari malformation

A 7 year old boy is taken by his parents to a dermatologist for nodules on his face. He has diagnosed with adenoma sebaceum. Common with which of the following diseases? A. Neurofibromatosis B. Sturge-weber syndrome C. Tuberous sclerosis D. Ataxia telangiectasia E. Fragile X syndrome

C. TS

A 29 year old woman has progressive gait disorder and dysmetria. Laboratory studies include a hematocrit of 55% and a routine urinalysis, which reveals excess protein and some RBCs in the urine. Urine culture is negative. The initial physical examination reveals an enlarged liver and spleen. Additional physical exam findings will most likely include which of the following? A. A Kayser-Fleischer ring around the cornea B. Hypopigmented (ash-leaf) spots on the trunk C. Telangiectasias in the fundi on retinal examination D. Bilateral hearing loss E. Generalized hyporeflexia

C. Telangiectasias - Erythocytosis + cerebellar signs + microscopic hematuria + hepatosplenomegaly suggests VHL - They also get retinal telangiectasias

A 62 year old man has started getting a haircut every week. Whenever he lays his head back to have his hair washed, he has the sensation of spinning. With vertigo that develops on extreme extension or rotation of the head, the patient probably has insufficiency in which of the following? A. Left subclavian artery B. Internal carotid arteries bilaterally C. Vertebrobasilar system D. Internal maxillary artery E. Innominate artery

C. Vertebrobasilar system - The vertebral arteries ascend through the foramens in the transverse processess of the cervical vertebrae - With bony spurs on the vertebrae or with severe atherosclerotic disease in the vertebral arteries, flow through the vertebrobasilar system may be reduced when the head is extended or rotated

A patient with suspected MS undergoes multimodality evoked potentials, EEG, MRI, and CSF testing. Which of the following evoked response patterns is most often abnormal in patients with early MS? A. Brainstem auditory evoked response B. Far-field somatosensory evoked response C. Visual evoked response D. Jolly test E. Sensory nerve conduction test

C. Visual evoked response - Optic neuritis appears early and often in many patients with MS - Components of the VER may be slowed or even absent - Just b/c this is messed up though doesn't guarantee a person has Ms - any disorder that causes optic neuritis can cause this

A 65 year old man has had disrupted cerebrospinal fluid flow for several years, secondary to a thoracic disk herniation. This abnormal physiologic state resulted in the formation of a cervical cystic lesion readily apparent on the MRI. Examination of this patient might reveal which of the following abnormalities? A. Third-nerve palsy B. Calf atrophy C. Charcot joints D. Atrophy of the intrinsic hand muscles E. Grasp reflexes

D. Atrophy of the intrinsic hand muscles - This is describing a syrinx - As this increases in size, it may affect the lower motor neuron in the anterior horn of the spinal cord, causing weakness, hypo/absent reflexes, atrophy, etc..

Within 6 years of his initial visit, a patient with VHL syndrome returns with a pathologic fracture of his spine. Biopsy reveals metastatic cancer. Whcih of the following is the likely source of the tumor? A. Cerebral hemisphere B. Cerebellar hemisphere C. Liver D. Kidney E. Spleen

D. Kidney - VHL is associated with a high incidence of renal carcinomas - These malignant tumors develop years after the cerebellar hemangioblastomas, liver disease, or PCKD - People who survive this initial disease often succumb to metastatic renal carcinoma

Which of the following is teh most definitive test for identifying intracranial aneurysms? A. MRI scanning B. CT scanning C. Single photon emission computed tomography D. Positron emission tomography E. Cerebral angiography

E. Cerebral angiography - CT is only good for ruptured aneurysms causing a bleed, not unruptured aneurysms

A 16 year old girl with complex partial seizures and mild mental retardation has a birthmark consisting of deep red discoloration extending over her forehead and left upper eyelid. A CT scan of her brain would be likely to reveal which of the following? A. A hemangioblastoma B. A Charcot-Bouchard aneurysm C. An arteriovenous malformation D. A leptomeningeal angioma E. A fusiform aneurysm

D. A leptomeningeal angioma - Sturge weber syndrome: facial cutaneous angiomas, leptomeningeal angiomas, port-wine stains

A 26 year old graduate student presents with confusion and mild right hemiparesis developing over the course of an evening. His girlfriend relates that he has been having severe headaches every morning for the past 2 weeks. While being evaluated in the emergency room, he has a generalized tonic-clonic seizure. When examined 2 hours later, he is lethargic and unable to recall recents events, has difficulty naming, and has a right pronator drift. There is mild weakness of abduction of the eyes bilaterally. Funduscopic examination might be expected to show which of the following? A. Pigmentary degeneration of the retina B. Hollenhorst plaques C. Retinal venous pulsations D. Blurring of the margins of the optic disc E. Pallor of the optic disc

D. Blurring of the margins of the optic disc - His symptoms are concerning for a tumor - The first signs of increased ICP in an eye exam is blurring of the margins of the optic disc

What part of the brain is affected in Parkinson's? A. Cerebral cortex B. Peripheral nerves C. Cerebral white matter D. Brainstem nuclei E. Cerebellum

D. Brainstem nuclei - remember, dipigmentation of the nuclei is substantia nigra

A 36 year old man is being evaluated for left-hand weakness. On examination, it is readily apparent that he has atrophy of the first dorsal interosseous muscle. This may indicate damage to which of the following spinal roots? A. C5 and C6 B. C6 and C7. C. C7 and C8 D. C8 and T1 E. T1 and T2

D. C8/T1 - Dorsal interossei muscles are all ulnar nerve - C8/T1

A 62 year old male with hypertension has an episode in which he suddenly loses vision in his left eye. He is outside walking up the street, as he does every day, when suddenly the vision in his left eye goes black. When he closes his right eye, he can barely see at all. Within 2 hours, the vision is back to normal. what is the next best step to assess this patient's risk of having another attack of this kind? A. Transthoracic echocardiogram B. Brain MRI C. Brain CT D. Carotid ultrasound E. Lumbar puncture

D. Carotid ultrasound - Emboli from the carotid artery may exit to the opthalmic artery and cause obstruction - Transient ischemia that occurs before the embolus breaks up usually produces transient vision loss in the ipsilateral eye

A 60 year old man comes to the offie for evaluation of gait imbalance, which he has had for th epast 3 days. The patient was hospitalized 2 weeks ago for a complicated enterococcal diabetic infection of the left foot. He was prescribed ampicillin and gentamicin, which he has continued via home IV antibiotics.The patient subsequently developed a constant sensation of objects moving around in his visual field when looking in any direction which has caused him to feel unsteady. He has no associated N//V. The patient has a history of poorly controlled T2DM and hypertension. His other medications include lisinopril and basl-bolus insulin. Temperature is 98, blood pressure is 140/96, pulse is 80 and respirations are 16. BMI is 34. The left foot wound is healing without discharge or surrouding erythema. Neuro exam shows 5/5 muscle strenght and 2+ reflexes in all extremities. Hearing is decreased in the right ear on the whisper test. Rapid head movement away from a fixed target causes eye movement away from the target followed by a horizontal saccade back toward it. Whcih of the following is the most likely cause of this patient's current symptoms? A. BPPV B. Cerebellar infarction C. Diabetic autoimmune dysfunction D. Drug toxicity E. Meniere disease F. Vertebrobasilar insufficiency

D. Drug toxicity - Aminoglycoside ototoxicity

A 25 year old woman presents after an episode of double vision. she reports waking up 2 months ago with new double vision, with each image side-by-side. She made this appointment at that time but the double vision resolved in two weeks and is now back to normal. She denies any headache, eye pain, visual field or acuity disturbance in either eye. She takes no medications and has no other medical problems. On examination, she has a right pronator drift and an up going left toe. MRI of the brain demonstrates two periventricular white matter lesions and a left pons hyper-intensity on T2-weighted imaging without any associated enhancement. What is the next therapeutic step? A. Natalizumab B. Plasma exchange C. Methylpredinosolone D. Glatiramer acetate E. IV Ig

D. Glatiramer acetate - Presentation suggests MS - However, she is not having an acute exacerbation, so you don't give her corticosteroids - Treatment of choice is going to be a disease modifying agent such as glatiramer acetate - Natalizumab is very effective, but is associated with the development of PML, so is usually reserved for refractory disease or very aggressive disease

A 39 year old woman was involved in a head-on collision at approximately 40 miles per hour. She was wearing her seat belt, but still sustained a cervical cord injury from hyperflexion and extension. A cervical syrinx is likely to evolve in this patient if there has been which of the following? A. Intraspinal hyperthermia B. Intraspinal hypothermia C. Intraspinal transient ischemia D. Intraspinal contusion E. intraspinal demyelination

D. Intraspinal contusion - After cervical cord contusion, cyst formation may occur as damaged tissue is removed

38 year old man, poorly controlled HIV. Presents with confusion and double vision. Has had headaches, nausea, and vomiting. Lethargic and disoriented. Pupils are equal and reactive, but the left eye does not move laterally with leftward gaze. Skin exam shows scattered, small, umbilicated papules with surrounding erythema. CT scan of the head reveals mildly enlarged ventricles but no hemorrhage, infarction, or mass lesions. What is the next best step in evaluation? A. Anti-Toxo antibody assay B. Empiric TMP-SMX C. Gadolinium-enhanced brain MRI D. Lumbar puncture and India ink stain E. Serum PCR for cytomegalovirus

D. LP and India Ink stain - This is Cryptococcus - Can obstruct and caused increased CSF< compressing the abducens nerve and causing the eye finding seen here - Skin findings commonly look like molluscum contagiosum A and B are toxo (would see this on brain imaging)

A 70 year old man with a history of hypertension and diabetes presents with sudden onset of slurred speech and difficulty swallowing. his examination is significant for marked dysarthrtia and limb ataxia affecting the right arm and right facial weakness. Which of the following is the most likely location of the lesion? A. Right caudate B. Left occipital lobe C. Left thalamus D. Left pons E. Amygdala

D. Left pons - Classic lacunar stroke: dysarthria-clumsy hand syndrome - Usually a stroke in the contralateral pons or internal capsule

Small fiber neuropathies are characterized by which of the following? A. Loss of reflexes B. Diminished muscle bulk C. Loss of propioception D. Loss of temperature sensation E. Decreased muscle strength

D. Loss of temperature sensation

A 32 year old female comes to the office with several days of tingling and numbness in both hands. She moved to Arizona in June to care for her ailing mother who died 2 weeks ago of metastatic lung cancer. Since moving, she has had occasional headaches, dizziness, fatigue, poor sleep and blurry vision. She has had no fevers, weight loss, or anorexia. Her medical history is unremarkable. The patient drinks 1-2 glasses of wine daily but does not use tobacco or illicit drugs. Her blood pressure is 132/70, and pulse is 78. Sensation to light touch and pain is decreased distally in the bilateral upper extremities. Muscle strength is 5/5 in the upper and lower extremities and deep tendon reflexes are normal. On funduscopic examination, the right optic disc appears hyperemic and swollen. which of the following is the best next step in management of this patient? A. Carotid Doppler ultrasound B. Cervical spine x-ray C. lumbar puncture D. MRI of the brain and spine E. Nerve conduction studies F. Serum immunoglobulin levels G. Trial of lorazepam

D. MRI of the brain and spine - This is MS - Symptoms can worsen with exposure to high temperatures, such as moving to Arizona in June (Uhthoff phenomenon)

A 12 year old boy is involved in a MVA in which he fractures his humerus. As the pain subsides he notes weakness of flexion at the elbow and numbness/paresthesias along the lateral aspect of his forearm. what nerve appears to have been affected? A. Radial nerve B. Axillary nerve C. Suprascapular nerve D. Musculocutaneous nerve E. Median nerve

D. Musculocutaneous nerve

A 20 year old woman comes to the office due to hearing loss. The patient is a student and has difficulty hearing her professors, even when sitting in the front row. She also has trouble with balance poses during yoga. The patient has no medical conditions and takes only oral contraceptives. She drinks alcohol on the weekends and does not smoke. her mother also developed hearing loss in her 20s. Ear examination is normal. Gait is normal, but standing on 1 foot is normal, especially with eyes closed. Audiogram shows bilateral sensorineural hearing loss. Which of the following is the most likely cause? A. Congenital long QT syndrome B. Mitrochondrial disorder C. Multiple sclerosis D. Neurofibromatosis E. Ostosclerosis

D. Neurofibromatosis - NF2 - bilateral vestibular schwannomas

A 72 year old man is brought to the office by his niece due to medication non-adherence. The patient has type 2 DM that has been well controlled wtih a basal-bolus insulin regimen, but recently he has not been taking his insulin regularly and his home glucometer readings have been high. In the past, he self-administered his insulin, but now he no longer knows how to. The patient's niece says he has been "letting himself go" around the house since his wife's death from cancer 3 years ago. He is also reluctant to leave the house after a fall several months ago and his family has had to help him with grocery shopping and other common errands. On physical exam, the patient appears unkempt and smells of urine. The lungs are clear to auscultation and heart sounds are normal. The abdomen is soft and nontender. Neurological examination reveals absence of bilateral ankle reflexes, and the patient's gait is slow with a wide base. Brief cognitive assessment reveals short-term memory impairment. HgBA1c is 10.2%. Which of the following is the next best step in management of this patient? A. Arrange placement in a nursing home B. Begin therapy for depression-related cognitive impairment C. Change antidiabetic therapy to oral medications D. Obtain MRI of the brain E. Provide diabetes education and reinforce compliance

D. Obtain MRI of the brain - This is NPH

During the initial treatment of a patient with status epilepticus, a nurse reports that the patient has just lost bladder control and that the urine appears darker than normal. The responsible physician examines the bed sheets and agrees with the nurse's assessment. The physician should immediately institue which of the following? A. Order placement of an indwelling urinary catheter B. Order methacholine to regular bladder emptying C. Request a surgical consulttion in anticipation of an exploratory laparatomy D. Order placement of a condom catheter E. Repeat a urologic consultation to assess the incontinence

D. Order placement of a condom catheter - Urinary incontinence is an expected consequence of status epilepticus and consequently should not raise concern for abdominal or other disturbances - Keeping the patient dry is important because of the risk of skin breakdown - Indwelling catheter is unnecessary and increases the risk of urinary tract infection

A 45 year old left-handed man has had recurrent attacks of "dizziness." He descirbes the sensation of feeling the room spinning. The episodes occur abruptly and usually last for approximately 45 minutes. The dizziness occurs about once per month, but may happen more frequently. Which of the following most accurately describes the early hearing loss in this disease? A. Overall frequencies B. Primarily over high frequencies C. Primarily over middle frequencies D. Primarily over low frequencies E. In virtually no patients

D. Primarily over low frequencies - Unlike presbycusis, lower teons are most susceptible to impaired perception during the initial phases of Meniere disease

A 40 year old man returns home from a party and falls asleep in a chair. Upon awakening he notes weakness and numbness in his right hand. Examination reveals moderate weakness of the brachioradialis muscle and an inability to extend the wrist and fingers. Deltoid, biceps, and triceps stregnth are normal. There is sensory loss to pinprick over the dorsal right hand. where is the lesion? A. Right median nerve at the antecubital fossa B. Right median nerve at the carpal tunnel C. Right radial nerve at the axilla D. Right radial nerve at the humerus E. The ulnar nerve at the elbow

D. Right radial nerve at the humerus

A 28 year old man who recently immigrated from Brazil presents with 3 months of fluctuating but slowly progressive bilateral lower extremity weakness, a little worse on the left side than on the right. After a complete evaluation, a parasite is diagnosed as the etiology. This organism's ova usually damage the nervous system at which level? A. Cerebrum B. Cerebellum C. Basal ganglia D. Spinal cord E. Peripheral nerves

D. Spinal cord - Schistosoma mansoni - The fluke itself does not invade the spinal cord, but it deposits its eggs in the valveless veins of Batson, which drain the intestines and communicate with the drainage from the lumbosacral spinal cord - The patient develops granulomas around the ova that lodge in the spinal cord, and these granulomatous lesions crush the cord

A 62 year old man has been diagnosed with an abdominal aortic aneurysm. He is told that he is at high risk for aneurysm rupture, which would almost certainly kill him. Although a surgical procedure could dramatically reduce this risk, the operation itself has risks, including postoperative paraplegia. The arteria radicularis magna (artery of Adamkiewicz) enters at approximately what level? A. C2 to C5 B. C5 to C8 C. T2 to T8 D. T10 to L1 E. L4 to S4

D. T10 to L1 - The Artery of Adamkiewicz is a major radicular artery and may supply the lower two-thirds of the spinal cord - It is at a risk of occlusion during abdominal aortic aneurysm repair

A 68 year old man with MG is evaluated for progressive weakness while hospitalized. The patient was admitted 2 days prior for fever, cough, and pleuritic chest pain. Chest x-ray demonstrated a right lower lobe consolidation for which the patient has been receiving IV ceftriaxone and azithromycin. He also has been receiving his outpatient dose of pyridostigmine. In the past several hours, he has experienced progressive generalized weakness and an inability to cough out sputum. His temperature is 99.8, blood pressure is 130/70, pulse is 110 and respiratory rate is 25. Oxygen saturation is 89% on 4 L/min oxygen by nasal cannula. The patient appears to be in distress and his breathing pattern is rapid and shallow with occasional gurgling sounds. Lung auscultation reveals coarse crackles throughout. There is mild weakness of the extremities, but deep tendon reflexes are normal. His vital capacity is no 1.0 K compared to 1.5 L at the time of admission, and arterial blood gas shows pH 7.27, CO2 55, and O2 60. The patient is intubated and moved to the intensive care unit. Which of the following is the best next step in management? A. Close observation only B. Increase pyridostigmine dose C. IV atropine D. Therapeutic plasma exchange E. Thymectomy

D. Therapeutic plasma exchange -This is a myasthenic crisis - Severe muscle weakness leading to respiratory failure, can be precipitated by infection or surgery - They need IV immunoglobulins or plasmapharesis plus corticosteroids

A 27 year old woman presents with pain and weakness of the left upper extremity. Her symptoms started 5 days ago with severe pain around the left shoulder and gradual weakness in the arm. She recalls a recent vaccination in that arm and was painting a room the week before. Her examination is remarkable for weakness of the left deltoid, biceps, and triceps muscle with loss of biceps and triceps reflexes. motor strength and deep tendon reflexes are normal in the right upper extremity with both lower extremities. Her symptoms are most consistent with which of the following disorders? A. ALS B. Carpal tunnel syndrome C. AIDP D. Idiopathic brachial plexopathy E. Subclavian steal syndrome

D. idiopathic brachial plexopathy - Also called Parsonage-Turner syndrome - Acute abrupt onset of shoulder pain, with gradual weakness to follow

An 18 year old man presents with sudden profound generalized weakness after playing a long and vigorous game of basketball. After the game, he and his teammates went out for an "all you can eat" pizza dinner. Nothing significant is noted on history except he has had similar, but milder, episodes in the past after exercise and large meals. Which of the following tests would be most useful in diagnosing this patient? A. Acetylcholine receptor antibodies B. Antinuclear antibodies C. Serum calcium D. serum potassium E. Serum thyroxine

D. serum potassium - Periodic paralysis - Hypokalemia and hyperkalemic forms - Hypokalemic forms are associated with high carbohydrate meals, resulting in low potassium levels in serum and clinical improvement with potassium replacement

A 1 year old girl is evaluated for developmental delay. Examination and testing reveals that she is having hundreds of seizures per day. The clinical manifestations are somewhat subtle and consist of sit-up like movements. The EEG shows multifocal, high-amplitude spikes and slowing. Which of the following is the treatment of choice for this patient? A. Carbamazepine B. Phenobarbital C. Phenytoin D. Divalproex sodium E. Adrenocorticotropic hormone

E. ACTH - This is a child with tuberous sclerosis

A 65 year old man is having a neurological examination becasue of tingling in his feet. During the course of the examination, it is noticed that pupillary constriction occurs with attempted adduction of the globe. This suggests which of the following? A. Mesencephalic infarction B. Pontine glioma C. Acute glaucoma D. Iridocyclitis E. Aberrant third-nerve regeneration

E. Aberrant third-nerve regeneration - Oculomotor fibers that have been damaged may reversibly regenerate and connect to the wrong target - This aberrant regeneration is seen most often with lesions that chronically compress the third nerve F

A 66 year old woman presents wtih fever and a generalized convulsion. MRI shows high T2 signal in the medial temporal lobes. Lumbar puncture is performed and routine CSF analysis indicates 100 lymphocytes, 15 red blood cells, xanthochromia, and a midlly elevated pressure. Which of the following is the most appropriate treatment for this patient? A. Dexamethasone B. Amphotericin B C. Gamma globulin D. Methotrexate E. Acyclovir

E. Acyclovir - This presentation suggests a viral encephalitis - Herpes simplex is the most common cause of this - Acyclovir IV for at least 14-21 days

A 21 year old male graduate student presents to the opthalmology clinic with 2 days of visual loss in his right eye. he had lost vision in the other eye 1 year previously, with recovery to about 90%. His past medical history is otherwise unremarkable. He denies other episodes of nervous system dysfunction but on review of system admitted to urinary urgency and a buzzing sensation in his right leg. On examination, his pupils are symmetrical and react briskly to light, but the right pupil dilates when the light is moved quickly from the left pupil to the right. Visual acuity is 20/30 left eye and 20/200 right eye. Visual fields to confrontation appears intact; curosry neurological examination in the eye clinic is reported as normal. Long-term treatment will most likely be influenced by the results of which of the following antibody titer laboratory tests? A. Anti-acetylcholine receptor B. Thyroid peroxidase antibody C. Antinuclear antibody D. Anti-Ro antibody E. Aquaporin-4- antibody

E. Aquaporin-4 antibody - Dont' be tricked - a presentation like this isn't always MS - Bilateral optic dysfunction with significant vision loss and spinal cord symptoms is more fitting with neuromyelitis optica - Neuromyelitis optica associated with aquaporin-4-antibodies is more aggressive and orten requires more aggressive immunosuppressive therapy than what is needed for MS

An otherwise healthy young woman has poorly responsive pupils that are dilated. Visual acuity is normal. A careful neurologicla examination reveals bilatearlly absent Achilles tendon jerks. Which of the following is the most likely diagnosis? A. A cervical spinal cord tumor B. a brainstem glioma C. MS D. A posterior communicating artery aneurysm E. Benign tonic pupillary dilation

E. Benign tonic pupillary dilation - This is called Adie tonic pupil - Usually seen in otherwise healthy young women and may occur in isolation or in association with absent tendon reflexes - Local trauma to the eye shoudl be considered if only one pupil is affected - If both pupils are affected, drug use should be considered

A 33 year old operating room nurse accidentally has blood splashed in her eyes during a proceudre. The surgical resident who examined her immediatley afterwards notices she has a 2-mm aniscoria and sends her immeidately to the emergency room. She feels well, is alert and talkative, and has no motor dysfunction. On examination, the emergency room physician recognizes that the iris of the eye with the smaller pupil is pale blue, whereas that of the other eye is brown. Which of the following is the most likely etiology of this women's aniscoria? A. Conjunctivitis B. Traumatic third nerve palsy C. Carotid artery dissection D. Pupillary sphincter injury E. Congenital

E. Congenital - Sympathetic innervation of the iris is required for the change in the color of the iris to occur after birth and infancy - Congenital horner syndrome may be inherited as an autosomal dominant trait - It is characterized by failure of one eye to develop normal iris color - Any injury to the eye after this early developmental period would not be expected to leave a difference in eye color from one side to another

A 36 year old woman has tunnel vision in which she reports the same size area of perception regardless of how far from the testing screen the examination is performed. This history often indicates which of the following? A. Retinitis pigmentosa B. Neurosyphillis C. Sarcoidosis D. Chorioretinitis E. Conversion disorder

E. Conversion disorder - Tunnel vision vs concentric constriction - Concentric constriction - area perceived enlarges as the test screen is moved farther away from the patient - can be seen in neurosyphillis - Tunnel vision, it's always the same size - is not suggestive of a characteristic pattern of visual field loss and should suggest either conversion disorder or malingering

The shortest life expectancy of patients with ALS is associated with which clincal sign? A. Atrophy of the interossei B. Atrophy of the gastrocnemius C. Fasciculations in the lumbrical muscles D. Atrophy of the pectoralis muscle E. Fasciculations of the tongue

E. Fasciculations of the tongue - Early involvement of musculature supplied by cranial nerves is a much graver prognosis than early limb involvement

Which one of these is a possible mechniasm of getting CJD? A. Sexual intercourse B. A blood tranfusion C. Consumption of raw fish D. An upper respiratory infection E. Growth hormone treatment

E. Growth hormone treatment - Can rarely be acquired from growth hormone preparations acquired from cadaver pituitary glands

A 24 year old woman comes to the ER with 2 days of increasing confusion. Last night her friends noticed she was acting "goofy" but thought she might have been drinking. This morning she walked into the wrong classroom and wouldn't leave stating "ive got to take this test!" On exam, she is alert and oriented to self, but thinks it is 2010 and that Obama is president. She avoids the light during testing of pupillary reflex and tends to flex her leg at the hip when you passively flex her neck. You notice a rash on her stomach that her friends think might be due to "grass burn" from flag football 4 days ago. She is becoming increasingly agitated during your evaluation. What is the next most appropriate step in management? a. IV lorazepam B. urine tox screen C. MRI brain D. Lumbar puncture E. IV antibiotics

E. IV antibiotics - Meningitis signs - more improtant to start antibiotics ASAP before MRI or lumbar puncture

The patellar tendon reflex involves sensory fibers of the femoral nerve that originate in which of the following spinal segments? A. S3 to S4 B. S2 to S3 C. S1 to S2 D. L4 to L5 E. L2 to L3

E. L2 to L3

A 25 year old man is being evaluated. Rhythmic jerk nystagmus is elicited by having the patient look at a rotating drug with strips on it. This finding suggests which of the following? A. Drug toxicity B. Brainstem ischemia C. Parinaud syndrome D. Unilateral parietal lobe damage E. No pathologic lesion in the brain

E. No pathologic lesion in the brain - This is called optokinetic nystagmus - Pattern of eye movements that should be elicitable with the normal patient - If the nystagmus is less obvious on rotating the drum in a given direction, the patient may have a pareital lesion responsible for the asymmetric response

A 29 year old man comes to the ED due to a week of progressive lower extremity weakness. He has no history of trauma or back pain. The patient had trigeminal neuralgia 3 months ago, and a self-limited upper respiratory illness 2 weeks ago. His temperature is 37, blood pressure is 122/76, and pulse is 82. Physical examination shows increased passive resistance to passive flexion and extension of the lower limbs. Deep tendon reflexes are 3+ and plantar reflexes are upgoing bilaterally. There is decreased vibratory and positional sensation in his left upper extremity but no other sensory loss. A lumbar puncture is performed. Which of the following is most likely found on the CSF analysis in this patient? A. Abnormal cytology B. Albuminocytologic dissociation C. Increased opening pressure D. Neutrophilic predominance E. Oligoclonal bands F. Positive VDRL test G. Presence of 14 3 3 protein

E. Oligoclonal bands - This is MS NOT GB - Patient has neurologic deficits separated in time and space A is a malignancy, B is Guillain-Barre, D is meningitis, F is neurosyphillis, G is CJ disease

A 72 year old woman is brought to the ED by her husband due to sudden-onset, right-sided weakness and numbness. The patient's husband notes that her symptoms have progressed over several minutes and were lateral accompanied by vomiting and headache. He also notes that she is now excessively somnolent. The patient has hypertension and persistent atrial fibrillation as well as myelodysplasia that has not required treatment. She takes amlodipine, metoprolol, warfarin and multivitamins. in the past couple of days, she has also used over-the-counter medications for cold symptoms. Temperature is 98.6m blood pressure is 172/90, pulse is 68 and respirations are 16. Which of the following is the next best step in management of this patient? A. Aspirin and labetalol B. Lumbar puncture C. Platelet transfusion D. Protamine sulfate E. Prothrombin complex concentrate F. Tissue plasminogen activator G. Tranexamic acid

E. Prothrombin complex concentrate - She was taking warfarin + a cold medicine which probably contained acetaminophen (potentiates the anti-coagulant effect of warfarin) - You need to reverse the warfarin - Vitamin K takes a couple hours, but PCC works in minutes

A 56 year old woman has been on dialysis for the past 10 years owing to chronic renal failure from chronic kidney disease. Which of the following is the most reliable treatment for the peripheral neuropathy associated with her condition? A. Thiamine supplements B. Clonazepam C. Phenytoin D. Minoxidil E. Renal transplant

E. Renal transplant -Thiamine is easily lost during dialysis and can be replaced by thiamine supplements; however, this is not nearly as effective as reversing the chronic neuropathy of renal failure by a renal transplant

A 61 year old man, who smokes five packs of cigarettes per day and has hypertension, had an abdominal aortic aneurysm repair 8 hours ago. The surgery went well and there were no reported perioperative complications. Now the patient is unable to move his legs and states that they are "numb." On examination, he has a flaccid paresis of both lower extremities and has impaired pinprick sensation to a T9 level bilaterally. Joint propioception is normal. Which of the following is the most likely diagnosis in this case? A. Cerebral stroke B. Conversion disorder C. MS D. Spinal cord compression E. Spinal cord infarct

E. Spinal cord infarct - Due to anterior spinal cord occlusion

A 19 year old man is brought to the ED due to a seizure. The patient's roommate was awakened by a noise 30 minutes ago and found him convulsing on the bed. Emergency medical services were called, and the patient had another brief seizure episode in the ambulance. He has no history of seizure or other medical conditions. The roommate says the patient went to bed late last night after a frat party. Temp is 99, blood pressure is 148/90 and pulse is 94. On physical examination, the patient is somnolent, groans to painful physical stimuli and does not follow instructions. There is a small tongue laceration. While being examined, the patient has another generalized tonic-clonic seizure, which terminates after administration of IV lorazepam. Which of the following is the most appropriate next step? A. Administer IV phenobarbital B. monitor with continuous EEG C. Perform lumbar puncture D. Obtain MRI of the brain E. start fosphenytoin infusion

E. Start fosphenytoin infusion - This is status epilepticus - You give a benzo to stop the seizure and a non-benzodiazepine seizure medication to prevent seizure recurrence

A 23 year old woman with a history of anorexia nervosa is evaluated due to acute-onset confusion while hospitalized. Over the past 6 months, she has been severely restricting her caloric intake and has lost 15.4 lb. On admission, she was tachycardic and orthostatic and her BMI was 15. The patient has been receiving IV hydration and parenteral nutrition. Today, her family notes that she seems newly confused and unsteady when walking. Temp is 98, blood pressure is 110/70, and pulse is 86. She is not oriented to time or place. Bilateral pupils are equal and reactive. Her lateral gaze is restricted on both sides and evokes a horizontal nystagmus. There is no nuchal rigidity or motor weakness. Bilateral ankle reflexes are diminished. The patient walks slowly with short and wide-based steps. Which of the following is the next best step in management of this patient? a) IM cobalamin b) IV acyclovir c) Parenteral phosphate d) Systemic steroids e) Thiamine supplement f) thrombolytic therapy

E. Thiamine supplement - Wernicke - Encephalopathy + oculomotor dysfunction + postural and gait ataxia

A 40 year old man has undergone genetic testing several years ago for an autosomal dominant condition, which has afflicted several members of his maternal family for several generations. The testing revealed that he has the defective gene, and he now believes that he is showing signs of the disease, including nonpurposeful movements of the extremities that are socially awkward and make daily activities more difficult. If this patient were to be exposed to L-dopa, which of the following would most likely be evoked? A. Generalized seizures B. Partial seizures C. Intention tremor D. Scanning speech E. Writhing and jerking movements of the limbs

E. Writhing and jerking movements of the limbs - He has huntingtons disease - Dopaminergic drugs unmask the chorea that is typical of this disease - Using this as a diagnostic technique is not advised because it may contribute to premature symptoms of chorea

A 64 year old woman comes to the office for evaluation of lower extremity weakness. The patient was hospitalized due to pneumonia and septic shock a month ago. She was intubated for respiratory failure at the time of admission and was treated in the ICU for several days. After adequate improvement in her respiratory status, she was discharged to a physical rehab facility. Since that time, the patient has had worsening lower extremity weakness that has failed to improve despite physical therapy. She also has had frequent occipital headaches and numbness and tingling in her hands and feet . The patient has a history of RA, hypertension, hypothyroidism, and right total knee replacement. She is a former smoker with a 20 pack year history and does not drink alcohol. Blood pressure is 136/82, pulse is 72. Physical examination shows chronic joint deformities in both hands, clear lungs, normal heart sounds, and a nontender abdomen. Neuro exam reveals decreased lower extremity muscle strength, hyperactive knee reflexes and decreased propioception in the feet. Rapidly flicking the nail elicits flexion of the ipsilateral thumb. Which of the following is the most likely cause of this patient's current symptoms? A. ACA occlusion B. critical illness related axonal injury C. immune mediated nerve demyelination D. inadequate thyroid hormone replacement E. instability of the atlantoaxial joint

E. instability of the atlantoaxial joint

An 8 year old boy presents to the ER with progressive clumsiness and headaches. He notes the headaches are worse in the mornings. An MRI performed in the ER shows an enhancing mass in the posterior fossa. Which is the most likely etiology of the mass? A. Choroid plexus tumor B. Germinoma C. Neurocytoma D. Lewy body disease E. Medulloblastoma

E. medulloblastoma - Medulloblastoma is the most common type of pediatric malignant tumor, orginating classically in the posterior fossa

Injuries to the macular or fovea centralis typically affect vision by producing which of the following? A. Bitemporal hemianopsia B. Nyctalopia C. Scintillating scotomas D. Mild loss of visual acuity E. Severe loss of visual acuity

E. severe loss of visual acuity - Injury to the macula usually results in severe loss of acuity with preservation of peripheral vision - Bitemporal - optic chiasm such as tumor - Nyctalopia - night blindness, retinal degeneration, vitamin A deficiency and color blindness - Scintillating scotomas - migraine with aura

A 58 year old woman comes to the ED due to several hours of left-sided headache, nausea and vomiting. She has blurred vision and sees halos around lights. The patient has a history of hand tremors for which she recently started taking a new medication. She previously smoked, but does not use alcohol or illicit drugs. Temp is 98.6, blood pressure is 150/90, pulse is 96. The patient is in severe distress. Physical examination reveals left eye crcum-corneal injection with a dilated and sluggish pupil. Visual acuity is decreased in the left eye. She has resting hand tremors and increased resistance to passive movements of the extremities. Which of the following medications is most likely causing her current symptoms? A. Entacapone B. Levodopa/carbidopa C. Pramiprexole D. Propranolol E. Selegiline F. Trihexyphenidyl

F. Trihexyphenidyl - Acute headache, nausea, blurry vision and a sluggish/dilated pupil - think acute angle-closure glaucoma - Most likely caused by trihexyphenidyl, anticholinergic used in Parkinson's - anticholinergics cause mydriasis which can precipitate the closure glaucoma

A 21 year old man is brought to the ED by his family after a witnessed generalized tonic clonic seizure. His sister says that the patient has never had a previous seizure but has had progressive headaches, fever, nasal congestion, and rhinorrhea over the last 3 weeks. He has no chronic medical problems, and does not use alcohol, tobacco or drugs. Temp is 102, blood pressure is 130/85, pulse is 96. The patient is drowsy and postictal. CT scan of the head with IV contrast reveals a 3-mm ring-enhancing lesion in the left frontal lobe. There are air-fluid levels and mucosal edema in the paranasal sinuses. Rapid HIV testing is negative. Tissue biopsy of the brain is likely to demonstrate which of the following? A. Herpes simplex virus B. Neoplastic cells C. Nocardia species D. Rhizopus species E. Toxoplasmosis F. Viridans streptococci

F. Viridans streptococci - Brain abscess, probably from sinusitis in this case - Staph, viridans strep, anaerobes - Nocardia can also cause brain abscess with ring-enhancing lesion but much more common in a severely immunocompromised person

A 64 year old woman is brought to the ED due to sudden-onset double vision for the past few hours. She has had no headache, vertigo, or slurring of speech and no focal weakness or numbness. The patient has a history of hypertension, type 2 DM with poor glycemic control, and chronic kidney disease. She smokes cigarettes but does not use alcohol or drugs. Blood pressure is 140/90 and pulse is 82. Physical exam shows ptosis of her right eye. The right eyelid is manually elevated, showing the right eye in a "down-and-out" position. Right eye adduction, elevation, and depression movements are significantly impaired. Left extreocular movements are normal in all directions. Bilateral pupils are equal and reactive to light. The remainder of the neuro exam is normal. Which of the following is the most likely cause of this patient's current condition? A. Brain demyelination B. Lacunar infarction C. Muscle infiltration D. Myoneural junction disorder E. Nerve compression F. Nerve inflammation G. Nerve ischemia

G. Nerve ischemia - She has signs of a oculomotor nerve palsy - Most common cause in adults is ischemic neuropathy due to poorly controlled diabetes

A 54 year old woman presents to your office complaining of difficulty walking. She describes severe weakness and occasional pain in her thigh muscles. She has stumbled and fallen several times over the last week. Her past medical history is significant for hypertension treated with hydrochlorothiazide and metoprolol. She consumes two to three cans of beer on weekends. Her younger brother died of a neurological disease when he was 20 years ago. Her mother suffers from hypertension and diabetes. Her heart rate is 90 and blood pressure is 170/100. Chest exam is normal. A bruit is heard over the left carotid artery. Neuro exam reveals hyporeflexia and decreased strength in all muscle groups. Her ESR is 12. ECG shows flat and broad T waves with occasional premature ventricular contractions. Which of the following is the most likely cause of this patient's complaints? A. Ischemic stroke B. Epidural hematoma C. subdural hematoma D. Lumar spinal stenosis E. MS F. ALS G. Thyroid myopathy H. Inflammatory myopathy I. Neuromuscular junction disease J. Electrolyte disturbance

J. Electrolyte disturbance - Hypokalemia can cause weakness, fatigue and muscle cramps + the flat T waves on the ECG


Related study sets

History 2020 Exam II Dr. Arthur Banton

View Set

Solar System - Chapter 9 - matching

View Set

AP Chem Unit 6 Missed Questions AP Classrm

View Set

Alterations of Fluid and Electrolyte Balance Quiz

View Set